Paeds Flashcards

1
Q

How does erythema toxicum neonatorum present?

A

aka newborn rash
around day 2-3, lasts about 24 hours
benign

white pinpoint papules at the centre of an erythematous base
looks like red blotches (baby acne) concentrated on trunk (and face)

How well did you know this?
1
Not at all
2
3
4
5
Perfectly
2
Q

How does strawberry naevus present?

A

aka cavernous hemangioma
grows until 6-9 months then regresses
benign

small flat red area > raised dimpled strawberry like lesion
mostly on head/neck

How well did you know this?
1
Not at all
2
3
4
5
Perfectly
3
Q

What tx can speed regression of a strawberry naevus?

A

topical propanolol

How well did you know this?
1
Not at all
2
3
4
5
Perfectly
4
Q

What is congenital dermal melanocytosis? How does it present?

A

mongolian blue spot

flat irregular blue/blue gray spots mostly on back/buttocks

can be mistaken for bruises
benign + spontaneous resolution

How well did you know this?
1
Not at all
2
3
4
5
Perfectly
5
Q

How does a stork bite/salmon patch present?

A

collection of blood vessels

pink patches on face/back of neck
blanch on pressure
get darker when crying/temp changes

fade over a few months

How well did you know this?
1
Not at all
2
3
4
5
Perfectly
6
Q

What is a caput succedaneum? What is it caused by?

A

boggy superficial scalp swelling that can cross suture line
(CA = CS - cap succ = cross suture)

caused by:
- pressure against cervix during birth (traumatic/prolonged birth)
- ventouse delivery

How well did you know this?
1
Not at all
2
3
4
5
Perfectly
7
Q

What is a cephalohaematoma? How is it managed?

A

subperiosteal haemorrhage due to damage to blood vessels during traumatic/prolonged/instrumental delivery

periosteum stuck tightly to skull = does not cross suture lines

usually does not require intervention
risk of anaemia + jaundice - monitor

How well did you know this?
1
Not at all
2
3
4
5
Perfectly
8
Q

What are the differentials for a superficial scalp swelling in a newborn? What is the possible complication of both?

A

caput succedaneum = crosses suture lines

cephalohaematoma = does not cross suture lines, more likely to cause discolouration of skin in area

jaundice = key complication > bruising and blood break down overwhelming newborn liver

How well did you know this?
1
Not at all
2
3
4
5
Perfectly
9
Q

When is neonatal jaundice abnormal?

A

very common

<24hrs old = always abnormal
2-14 days = normal
>2wks = can be either

How well did you know this?
1
Not at all
2
3
4
5
Perfectly
10
Q

Neonatal jaundice is caused by raised levels of what?

A

bilirubin

How well did you know this?
1
Not at all
2
3
4
5
Perfectly
11
Q

What are the causes of neonatal jaundice in neonates <24 hours old?

A

Sepsis :(
rhesus haemolytic disease
ACO incompatibility
TORCH infections
G6PD deficiency (x-linked recessive)
Hereditary spherocytosis (AD)
Cephalohematoma
Crigler-najjar syndrome (no UGT enzyme)

How well did you know this?
1
Not at all
2
3
4
5
Perfectly
12
Q

When does rhesus haemolytic disease occur?

A

mum is rhesus D -ve and baby is rhesus D +ve

How well did you know this?
1
Not at all
2
3
4
5
Perfectly
13
Q

What is the pathophysiology of rhesus haemolytic disease?

A

if mother is rhesus D -ve and baby is +ve
blood from the baby enters mother’s bloodstream > mother recognises baby’s rhesus D antigens on the rbcs as foreign and produces abs against them = mother is sensitised

sensitisation = usually fine in 1st pregnancy unless occurs early on
subsequent pregnancies = mother’s anti-D abs can cross placenta into fetus
if fetus is rhesus +ve > abs attach and cause immune system of fetus to attack own rbcs
= haemolysis > anaemia, high bilirubin

How well did you know this?
1
Not at all
2
3
4
5
Perfectly
14
Q

How does normal physiological jaundice occur in the newborn?

A
  1. high conc of rbc in fetus + neonate
  2. rbcs are more fragile (70 day lifespan) > release lots of bilirubin
  3. less developed liver function > normally excreted by placenta > no placenta > normal rise in bilirubin shortly after birth = mild yellowing of skin and sclera from 2-7 days
How well did you know this?
1
Not at all
2
3
4
5
Perfectly
15
Q

Causes of neonatal jaundice can be split into increased production and decreased clearance of bilirubin. Give some examples of each.

A

increased prod:
haemolytic disease, ABO incompat, sepsis, G6PD def, haemorrhage, cephalohaematoma

decreased clearance: prematurity, breast milk jaundce, cholestasis, extrahepatic biliary atresia, endocrine disorders, gilbert

How well did you know this?
1
Not at all
2
3
4
5
Perfectly
16
Q

What is kernicterus? How does it happen?

A

bilirubin can cross BBB > unconj bil deposition in abasal ganglia + brainstem after it exceeds the albumin binding capacity

excessive levels can lead to
brain damage

How well did you know this?
1
Not at all
2
3
4
5
Perfectly
17
Q

What are the causes of prolonged jaundice? When is jaundice considered prolonged?

A

> 14 days
21 days in a premature baby

biliary atresia
hypothyroidism
breast milk
CF
galactosaemia
UTI
gilbert syndrome

How well did you know this?
1
Not at all
2
3
4
5
Perfectly
18
Q

How is neonatal jaundice managed?

A

total bilirubin on tx threshold chart

phototherapy
severe jaundice - exchange transfusion

How well did you know this?
1
Not at all
2
3
4
5
Perfectly
19
Q

What is phototherapy? How does it correct jaundice?

A

light-box shines blue light on baby’s skin

converts unc bil into isomers that can be excreted in the bile/urine without needing conjugation in the liver

How well did you know this?
1
Not at all
2
3
4
5
Perfectly
20
Q

How does kernicterus present?

A

less responsive, floppy, drowsy baby with poor feeding

can cause seizures, hypotonia, opisthotonos

How well did you know this?
1
Not at all
2
3
4
5
Perfectly
21
Q

Is brain damage caused by kernicterus permanent?

A

yes

> cerebral palsy, LD, deafness

How well did you know this?
1
Not at all
2
3
4
5
Perfectly
22
Q

Why are babies who are breastfed more likely to become jaundiced?

A

components inhibit the ability of liver to process bilirubin
more likely to become dehydrated if not feeding adequately > slow passage of stools > increases absorption of bilirubin in intestines

How well did you know this?
1
Not at all
2
3
4
5
Perfectly
23
Q

How should neonatal jaundice be investigated?

A

FBC (anaemia, haemolysis)
blood film (G6PD, spherocytosis

unconjugated (extra-hepatic source)
conjugated (hepatobiliary source)

blood type of mother + baby (ABO/rhesus incompatability)
direct combs test (haemolysis)
thyroid function (hypo)
LFTs (hepatitis)

How well did you know this?
1
Not at all
2
3
4
5
Perfectly
24
Q

What is a tx threshold chart for neonatal jaundice?

A

age versus total bilirubin

How well did you know this?
1
Not at all
2
3
4
5
Perfectly
25
Q

How is haemolytic disease of the newborn prevented?

A

anti D immunoglobulin at 28 weeks + after sensitising events + 1 dose after birth if mother is rhesus neg

How well did you know this?
1
Not at all
2
3
4
5
Perfectly
26
Q

What bloods groups are mother and baby for ABO incompatability to occur?

A

when mother is type O and baby is type A or B

How well did you know this?
1
Not at all
2
3
4
5
Perfectly
27
Q

What are possible sensitisation events during pregnancy?

A

antepartum haemorrhage
amniocentesis procedures
abdo trauma

How well did you know this?
1
Not at all
2
3
4
5
Perfectly
28
Q

What is Kleinhauer test?

A

checks how much fetal blood has passed into the mother’s blood during a sensitisation event

How well did you know this?
1
Not at all
2
3
4
5
Perfectly
29
Q

What are the 4 ABO blood groups and their antigens?

A

A - A antigen
B - B antigen
AB - both A and B
O - no ABO antigens

How well did you know this?
1
Not at all
2
3
4
5
Perfectly
30
Q

When do abs to ABO antigens start to be produced in an individual?

A

individuals generate IgM ABs to antigens absent from their rbcs without prior exposure

begins at birth, detectable from 3 months ish

How well did you know this?
1
Not at all
2
3
4
5
Perfectly
31
Q

Which abs are produced in each ABO blood group?

A

A - anti-B
B - anti-A
AB - no ABO abs
O - anti-A, anti-B, anti-AB

How well did you know this?
1
Not at all
2
3
4
5
Perfectly
32
Q

Which blood types can each blood group receive? Which are the universal recipient and donor?

A

A - A, O
B - B, O
AB - A, B, AB, O (universal recipient)
O - O (universal donor)

How well did you know this?
1
Not at all
2
3
4
5
Perfectly
33
Q

Why does ABO incompatibility occur?

A

ABO Abs are typically IgM = does not cross placental barrier
Small number of people develop IgG ABO Abs > these mothers can have haemolytic disease of the fetus and newborn due to ABO incompatibility

How well did you know this?
1
Not at all
2
3
4
5
Perfectly
34
Q

Which type of ab can cross the placenta?

A

IgG only

How well did you know this?
1
Not at all
2
3
4
5
Perfectly
35
Q

What are the TORCH infections?

A

toxoplasmosis
other (syphilis, parvovirus, varicella zoster)
rubella
CMV
herpes/hepatitis/HIV

How well did you know this?
1
Not at all
2
3
4
5
Perfectly
36
Q

What can neonatal jaundice be exacerbated by?

A

polycythaemia (gestational diabetes)

bruising (caput succedaneum and cephalohaematoma)

How well did you know this?
1
Not at all
2
3
4
5
Perfectly
37
Q

What are the causes of (normal) jaundice between 2 and 14 days?

A

physiological
breast milk

infection > dehydration

How well did you know this?
1
Not at all
2
3
4
5
Perfectly
38
Q

Why can congenital hypothyroidism cause jaundice?

A

slows everything down > impaired bilirubin excretion

How well did you know this?
1
Not at all
2
3
4
5
Perfectly
39
Q

How is congential hypothyroidism found?

A

newborn blood spot screening test

How well did you know this?
1
Not at all
2
3
4
5
Perfectly
40
Q

How do babies with congenital hypothyroidism present?

A

prolonged neonatal jaundice
poor feeding
constipation
increased sleeping
reduced activity
slow growth + development

How well did you know this?
1
Not at all
2
3
4
5
Perfectly
41
Q

What two things can happen to the thyroid gland that causes hypothyroidism?

A

dysgenesis - underdeveloped gland

dyshormonogenesis - full developed but does not produce enough hormone

How well did you know this?
1
Not at all
2
3
4
5
Perfectly
42
Q

What is biliary atresia?

A

section of the bile duct is narrowed/absent

How well did you know this?
1
Not at all
2
3
4
5
Perfectly
43
Q

Why does biliary atresia cause jaundice?

A

> cholestasis > conj bilirubin still excreted into the bile > bile can’t be transported from liver to bowel and isn’t excreted > jaundice

How well did you know this?
1
Not at all
2
3
4
5
Perfectly
44
Q

How does biliary atresia present?

A

prolonged jaundice

How well did you know this?
1
Not at all
2
3
4
5
Perfectly
45
Q

What is the initial investigation for biliary atresia?

A

conjugated and unconjugated bilirubin

> high proportion of conjugated = liver processing it and not excreting it

How well did you know this?
1
Not at all
2
3
4
5
Perfectly
46
Q

How is biliary atresia treated?

A

surgery - attach small intestine to liver

often require a liver transplant eventually

How well did you know this?
1
Not at all
2
3
4
5
Perfectly
47
Q

Why babies are at higher risk of jaundice and kernicterus?

A

premature babies
due to immature liver

How well did you know this?
1
Not at all
2
3
4
5
Perfectly
48
Q

How does oesophageal atresia/tracheo-oesophageal fistula present?

A

polyhydramnios (cannot swallow AF)

blowing bubbles
salivation/drooling
cyanotic episodes on feeding
resp distress/aspiration

How well did you know this?
1
Not at all
2
3
4
5
Perfectly
49
Q

How is oesophageal atresia/tracheo-oesophageal fistula diagnosed?

A

pass NG tube and x-ray > NG tube will coil in oesophagus

abdo XR - no bubbles = isolated OA

How well did you know this?
1
Not at all
2
3
4
5
Perfectly
50
Q

What is gastroschisis?

A

paraumbilical defect (hole in the abdominal wall beside belly button) - always to right of the cord
intestinal contents often on outside of body

How well did you know this?
1
Not at all
2
3
4
5
Perfectly
51
Q

What is gastroschisis caused by?

A

likely a vascular accident in early embryonic life

How well did you know this?
1
Not at all
2
3
4
5
Perfectly
52
Q

How do gastroschisis and omphaloceles present on investigation?

A

antenatal scans usually
raised AFP

intestines likely to be outside of body at birth
may have comorbid intestinal atresia

How well did you know this?
1
Not at all
2
3
4
5
Perfectly
53
Q

How is gastroschisis treated?

A

requires LSCS delivery

staged corrective surgery
TPN slowly introduced

How well did you know this?
1
Not at all
2
3
4
5
Perfectly
54
Q

What is an omphalocele?

A

ventral defect in umbilical ring with herniation of abdominal contents covered by peritoneum
(similar to gastroschisis but covered by thin sac)

How well did you know this?
1
Not at all
2
3
4
5
Perfectly
55
Q

What can cause an omphalocele?

A

small defect - beckwith-wiedemann syndrome

large defects - pulmonary hypoplasia

How well did you know this?
1
Not at all
2
3
4
5
Perfectly
56
Q

How is an omphalocele treated?

A

may need LSCS if large
protect herniated contents, prevent hypothermia

gastric decompression
staged surgical repair

How well did you know this?
1
Not at all
2
3
4
5
Perfectly
57
Q

When is a congenital diaphragmatic hernia likely to present?

A

prenatal US

How well did you know this?
1
Not at all
2
3
4
5
Perfectly
58
Q

How does a CDH usually present?

A

newborn resp distress (may need resuscitation at birth)
if small - resp distress after feeding

displaced apex beat
bowel sounds in hemithorax
scaphoid abdomen

How well did you know this?
1
Not at all
2
3
4
5
Perfectly
59
Q

What are the causes of small bowel obstruction?

A

duodenal atresia
malrotation + volvulus
meconium ileus in CF

How well did you know this?
1
Not at all
2
3
4
5
Perfectly
60
Q

What condition is duodenal atresia a/w?

A

Down’s (1/3rd)

How well did you know this?
1
Not at all
2
3
4
5
Perfectly
61
Q

What XR is duodenal atresia likely to present with?

A

double bubble

How well did you know this?
1
Not at all
2
3
4
5
Perfectly
62
Q

What is a volvulus? When does it present?

A

loop of intestine twists round itself and the mesentery attached to it > bowel obstruction

How well did you know this?
1
Not at all
2
3
4
5
Perfectly
63
Q

When does a volvulus generally present?

A

presents later (2-30 days) so meconium will have passed normally

How well did you know this?
1
Not at all
2
3
4
5
Perfectly
64
Q

What XR sign does volvulus present with?

A

coffee bean sign

How well did you know this?
1
Not at all
2
3
4
5
Perfectly
65
Q

What is meconium ileus? What is it a/w?

A

pathognomonic for CF, usually the 1st sign

meconium thick + sticky > gets stuck and obstructs bowel

How well did you know this?
1
Not at all
2
3
4
5
Perfectly
66
Q

When should meconium be passed by? What should it look like?

A

24 hours
black

How well did you know this?
1
Not at all
2
3
4
5
Perfectly
67
Q

How does meconium ileus present?

A

not passing meconium within 24hrs
abdo distention
vomiting

SBO

How well did you know this?
1
Not at all
2
3
4
5
Perfectly
68
Q

What test should be done if a baby has meconium ileus?

A

heel prick for CF

How well did you know this?
1
Not at all
2
3
4
5
Perfectly
69
Q

How can meconium ileus be treated?

A

therapeutic contrast enema (gastrografin) to pass it

How well did you know this?
1
Not at all
2
3
4
5
Perfectly
70
Q

How can meconium ileus be differentiated from Hirschsprung’s after an enema?

A

will pass after enema in MI, not after hirschsprung

How well did you know this?
1
Not at all
2
3
4
5
Perfectly
71
Q

What is hirschsprung’s disease?

A

congenital condition where nerve cells of myenteric + submucosal plexus absent in a segment of the colon (usually rectosigmoid)

How well did you know this?
1
Not at all
2
3
4
5
Perfectly
72
Q

What is the pathophysiology of Hirschsprung’s?

A

in fetal development parasympathetic ganglion cells start high in GI tract and migrate down > in hirschsprung’s they don’t travel all the way so a section if left without them

responsible for stimulating peristalsis > bowel looses motility + stops passing food along

How well did you know this?
1
Not at all
2
3
4
5
Perfectly
73
Q

Does the aganglionic section of colon in Hirschsprung’s relax or constrict?

A

constricts > obstruction

proximal to obstruction > distended and full

How well did you know this?
1
Not at all
2
3
4
5
Perfectly
74
Q

How does Hirschsprung’s present?

A

functional LBO with failure to pass meconium within 24hrs
chronic constipation
abdo distention
late bilious vomiting
poor weight gain/failure to thrive

How well did you know this?
1
Not at all
2
3
4
5
Perfectly
75
Q

What would happen after a PR in a newborn with Hirschsprung’s?

A

contracted distal segment followed by rush of liquid stool and temporary relief of symptoms

How well did you know this?
1
Not at all
2
3
4
5
Perfectly
76
Q

How is a Hirschsprung’s investigated? What is the GS?

A

AXR - contracted distal segment + dilated proximal segment
barium enema
GS = rectal suction biopsy

How well did you know this?
1
Not at all
2
3
4
5
Perfectly
77
Q

What complication can be caused by Hirschsprung’s?

A

Hirschsprung-associated enterocolitis

fever
abdo distention
diarrhoea +/- blood
features of sepsis

= life threatening, made lead to toxic megacolon and perforation fo the bowel

How well did you know this?
1
Not at all
2
3
4
5
Perfectly
78
Q

How is Hirschsprung’s-associated enterocolitis treated?

A

abx
fluid resuscitation
decompression of obstructed bowel

How well did you know this?
1
Not at all
2
3
4
5
Perfectly
79
Q

How is Hirschsprung’s managed?

A

if unwell - fluid resus, mx of BO

surgical removal of aganglionic section = swenson procedure

How well did you know this?
1
Not at all
2
3
4
5
Perfectly
80
Q

What are the causes of intestinal obstruction?

A

meconium ileus
Hirschsprung’s
oesophageal/duodenal atresia
intussusception
imperforate anus
malrotation with a volvulus
strangulated hernia

How well did you know this?
1
Not at all
2
3
4
5
Perfectly
81
Q

What kind of vomiting does a bowel obstruction present with?

A

persistent bilious vomiting - contains bright green bile

How well did you know this?
1
Not at all
2
3
4
5
Perfectly
82
Q

What abnormal bowel sounds may be heard with a bowel obstruction?

A

high pitched
tinkling early on > absent later

How well did you know this?
1
Not at all
2
3
4
5
Perfectly
83
Q

How does a bowel obstruction generally present on XR?

A

dilated loops proximal to obstruction
collapsed loops distal
absence of air in rectum

How well did you know this?
1
Not at all
2
3
4
5
Perfectly
84
Q

What is the general management of a bowel osbtruction?

A

NBM
NG tube - drain stomach and stop vomiting
IV fluids

How well did you know this?
1
Not at all
2
3
4
5
Perfectly
85
Q

What are the RFs for undescended testes/cryptorchidism?

A

FHx
low birth weight
SGA
prematurity
maternal smoking

How well did you know this?
1
Not at all
2
3
4
5
Perfectly
86
Q

Undescended testes in older children/after puberty leads to a higher risk of what?

A

testicular torsion
infertility
testicular ca

How well did you know this?
1
Not at all
2
3
4
5
Perfectly
87
Q

How are undescended testes managed?

A

W&W in newborns - most will occur spontaneously at 3-6mo
longer it takes, less likely it will happen spontaneously

6mo > refer to urologist
orchidopexy = surgical correction at 6mo-1yr

How well did you know this?
1
Not at all
2
3
4
5
Perfectly
88
Q

When should testicular descent occur during pregnancy?

A

8th month
requires testosterone

How well did you know this?
1
Not at all
2
3
4
5
Perfectly
89
Q

What is the difference between indirect and direct hernias?

A

almost all indirect - through deep then superficial inguinal ring

direct - defect in posterior inguinal wall and passes through to superficial inguinal ring, in premature babies with weak wall

How well did you know this?
1
Not at all
2
3
4
5
Perfectly
90
Q

What is the cause of a neonatal inguinal hernia?

A

patent processus vaginalis - failure to become tunica vaginalis (serous membrane that covers the testes)

How well did you know this?
1
Not at all
2
3
4
5
Perfectly
91
Q

What are the signs of an inguinal hernia?

A

bulge lateral to pubic tubercle
more prominent on crying
most can be reduced

How well did you know this?
1
Not at all
2
3
4
5
Perfectly
92
Q

What is an incarcerated inguinal hernia? What are the signs? What is the risk?

A

contents of hernia become stuck in groin and can’t be reduced back into abdomen

tender lump
irritable
vomiting

risk of strangulation

How well did you know this?
1
Not at all
2
3
4
5
Perfectly
93
Q

What test can differentiate between a direct and indirect inguinal hernia?

A

ring occlusion test

How well did you know this?
1
Not at all
2
3
4
5
Perfectly
94
Q

How is an inguinal hernia managed?

A

always surgery due to risk of incarceration
emergency surgery if incarcerated

How well did you know this?
1
Not at all
2
3
4
5
Perfectly
95
Q

What is hypospadias?

A

presence of urethral meatus on ventral aspect of penile shaft

How well did you know this?
1
Not at all
2
3
4
5
Perfectly
96
Q

What should be considered if a child has hypospadias + undescended testes?

A

causes of low testosterone

testosterone needed for meatus to move distally

How well did you know this?
1
Not at all
2
3
4
5
Perfectly
97
Q

How do children with hypospadias present?

A

meatus on ventral surface
hooded foreskin
spraying on urination

How well did you know this?
1
Not at all
2
3
4
5
Perfectly
98
Q

How is hypospadias managed?

A

surgery not necessary unless parents want it
cannot have circumcision as skin needed

How well did you know this?
1
Not at all
2
3
4
5
Perfectly
99
Q

What is testicular torsion?

A

twisting of spermatic cord cutting off blood supply to testicle

How well did you know this?
1
Not at all
2
3
4
5
Perfectly
100
Q

What are the RFs for testicular torsion?

A

high testosterone - larger testes
neonates + pubertal teenagers
undescended testes
bell clapper deformity

How well did you know this?
1
Not at all
2
3
4
5
Perfectly
101
Q

How does testicular torsion present?

A

testicle higher and more horizontal
acutely swollen and tender
vomiting due to pain
absent cremasteric reflex (stroke inner thigh, testicle doesn’t elevate)
negative prehn’s sign (pain not relieved on elevation)

How well did you know this?
1
Not at all
2
3
4
5
Perfectly
102
Q

How can testicular torsion and epididymitis be differentiated?

A

prehn’s sign

+ve = pain relieves on elevation = epididymitis
-ve = pain not relieved = torsion

How well did you know this?
1
Not at all
2
3
4
5
Perfectly
103
Q

How is testicular torsion managed?

A

surgical emergency - within 6 hours

after 24hrs very poor chance of testicular survival

How well did you know this?
1
Not at all
2
3
4
5
Perfectly
104
Q

What is intussusception?

A

a section of the bowel telescopes into another
usually ileum into caecum

How well did you know this?
1
Not at all
2
3
4
5
Perfectly
105
Q

What conditions is intussusception a/w?

A

often preceded by a viral illness

resp: CF
gastro: intestinal polyps, meckel diverticulum
lymphoma, henoch-schonlein purpura

How well did you know this?
1
Not at all
2
3
4
5
Perfectly
106
Q

How does intussusception present?

A

severe colicky abdo pain, legs draw up
pale, lethargic unwell child
signs of SBO (vomiting, constipation, distention)

sausage shaped mass in abdomen (often RUQ)
redcurrant jelly stool (due to bowel ischaemia = late sign)

How well did you know this?
1
Not at all
2
3
4
5
Perfectly
107
Q

How is intussusception investigated?

A

USS - target/donut sign
AXR - dilated proximal bowel loops

How well did you know this?
1
Not at all
2
3
4
5
Perfectly
108
Q

How is intussusception managed?

A

rectal air insufflation (pump air into colon to push it out)

surgical correction if fails/signs of ischaemia

How well did you know this?
1
Not at all
2
3
4
5
Perfectly
109
Q

What is pyloric stenosis?

A

hypertrophy of the pylorus preventing food traveling from the stomach to the duodenum

How well did you know this?
1
Not at all
2
3
4
5
Perfectly
110
Q

What age is intussusception likely to happen?

A

3mo - 2yrs

How well did you know this?
1
Not at all
2
3
4
5
Perfectly
111
Q

How does pyloric stenosis present?

A

projectile non bilious vomiting shortly after feeding
thin pale hungry baby, failing to thrive
first few weeks of life
dehydration

firm round mass in upper abdomen
hypochloric hypokalaemic metabolic alkalosis

How well did you know this?
1
Not at all
2
3
4
5
Perfectly
112
Q

How is pyloric stenosis investigated?

A

test feed - feel for olive size mass in RUQ
USS - thickened pylorus
ABG - hypochloric hypokalaemic metabolic alkalosis

How well did you know this?
1
Not at all
2
3
4
5
Perfectly
113
Q

What is necrotising entercolitis?

A

inflammatory bowel necrosis

How well did you know this?
1
Not at all
2
3
4
5
Perfectly
114
Q

What are RFs for NEC?

A

PREMATURITY - resp distress + hypoxia

indomethacin (NSAID) for PDA > bowel ischaemia (causes vasoconstriction of mesenteric arteries)

How well did you know this?
1
Not at all
2
3
4
5
Perfectly
115
Q

How does NEC present?

A

feed intolerance
vomiting +/- bile staining
PR fresh blood + mucus
abdominal distention with taught shiny skin

How well did you know this?
1
Not at all
2
3
4
5
Perfectly
116
Q

How is NEC diagnosised?

A

FBC - low platelets = poor prognosis
blood cultures
clotting screen - can lead to DIC

AXR: pneumatosis intestinalis/intramural air = pathognomonic, dilated bowel loops

How well did you know this?
1
Not at all
2
3
4
5
Perfectly
117
Q

How is NEC treated?

A

stop oral feeds
cefotaxime + vancomycin

laparotomy if rapid distention + sigs of perforation

How well did you know this?
1
Not at all
2
3
4
5
Perfectly
118
Q

What is GORD?

A

contents of stomach reflux through the LOS into the oesophagus, throat and mouth

How well did you know this?
1
Not at all
2
3
4
5
Perfectly
119
Q

Why are babies prone to GORD?

A
  1. immaturity of the LOS
    normal for a baby to reflux feeds - most stop spontaneously after 1yr
  2. mostly liquid diet
  3. mostly horizontal position
How well did you know this?
1
Not at all
2
3
4
5
Perfectly
120
Q

What is the most common cause of vomiting in infants?

A

GORD

How well did you know this?
1
Not at all
2
3
4
5
Perfectly
121
Q

What vomiting occurs with GORD?

A

persistant vomiting, NO bile

How well did you know this?
1
Not at all
2
3
4
5
Perfectly
122
Q

How is GORD diagnosed?

A

clinically usually

red flags eg bilious vomiting > referral to paeds
severe sx > 24hr pH monitoring

How well did you know this?
1
Not at all
2
3
4
5
Perfectly
123
Q

How is GORD treated?

A
  1. reassurance, smaller more frequent feeds, sit upright after feeds, thickening agents eg carobel for bottles, gaviscon (alginate therapy)
  2. PPI or H2 receptor antagonist
    D2 antagonist eg domperidone (enhance gastric emptying) could also be tried
  3. unresponsive to tx + >1yr: nissen fundoplication
How well did you know this?
1
Not at all
2
3
4
5
Perfectly
124
Q

How can vomiting due to GORD vs CMPA vs intestinal obstruction vs infection be differentiated?

A

GORD - chronic, recurrent regurg, feeding difficulty, arching of back/neck, sore throat

CMPA - chronic, abdo pain, eczema, flatulence, bloody stools, diarrhoea/constipation

obstruction - acute on chronic bilious vomiting, constipation, abdo pain

infection - acute D&V + sx of localised infection eg abdo pain in GE

How well did you know this?
1
Not at all
2
3
4
5
Perfectly
125
Q

What are the two types of CMPA? How can they be differentiated by the timing of their sx?

A

IgE mediated - type 1 hypersensitivity > immediate sx onset after only small amounts

Non-IgE mediated > sx occur about 72hrs after ingestion

How well did you know this?
1
Not at all
2
3
4
5
Perfectly
126
Q

What are the clinical features of IgE mediated CMPA?

A

urticaria, pruritus
colicky abdo pain, vomiting, bloody stool
rhinorrhea, itchy nose

How well did you know this?
1
Not at all
2
3
4
5
Perfectly
127
Q

What are the clinical features of non-IgE mediated CMPA?

A

loose frequent stool with blood + mucus
colicky abdo pain

How well did you know this?
1
Not at all
2
3
4
5
Perfectly
128
Q

How is CMPA diagnosed?

A

IgE > skin prick (4mm = +ve)
Non IgE > temporary removal from diet then reintroduction to see if sx return

How well did you know this?
1
Not at all
2
3
4
5
Perfectly
129
Q

What is sandifer syndrome?

A

brief episodes of abnormal movements a/w GORD in infants
otherwise neurologically normal
usually resolves as reflux improves

torticollis - forceful contraction of neck muscles causing twisting of neck
dystonia - abnormal muscle contractions causing twisting movements - arching back, unusual posture

How well did you know this?
1
Not at all
2
3
4
5
Perfectly
130
Q

How are breastfed and bottle fed babies tx for CMPA?

A

breastfed > remove milk from mum’s diet
bottle > extensively hydrolysed formula

How well did you know this?
1
Not at all
2
3
4
5
Perfectly
131
Q

What is functional constipation?

A

no significant underlying cause except simple lifestyle factors
number of times someone opens their bowels varies between individuals eg breastfed babies generally open them much less

How well did you know this?
1
Not at all
2
3
4
5
Perfectly
132
Q

What are the typical features of constipation in a child?

A

<3 stools/week
hard rabbit dropping stools that are difficult to pass + a/w straining, pain and bleeding
retentive posturing
abdo pain that waxes and wanes
avoidance of eating due to fear of pain

How well did you know this?
1
Not at all
2
3
4
5
Perfectly
133
Q

What can happen to bowel movements in chronic constipation?

A

overflow soiling - rectum distends leading to loss of sensation > incontinence with loose smelly stools

How well did you know this?
1
Not at all
2
3
4
5
Perfectly
134
Q

When is faecal incontinence/encopresis considered pathological?

A

4 yrs
usually due to chronic constipation where the rectum has stretched and lost sensation > loose stools bypass blockage and leak out

How well did you know this?
1
Not at all
2
3
4
5
Perfectly
135
Q

What are some pathological causes of encopresis?

A

spina bifida
hirschprung’s disease
cerebral palsy, LD, psychosocial stress/abuse

How well did you know this?
1
Not at all
2
3
4
5
Perfectly
136
Q

What lifestyle factors can contribute to constipation?

A

habitually not opening bowels
low fibre diet
poor fluid intake and dehydration
sedentary lifestyle
psychosocial problems

How well did you know this?
1
Not at all
2
3
4
5
Perfectly
137
Q

What are some secondary causes of constipation?

A

hirschsprung’s disease
CF - meconium ileus
IO
anal stenosis

hypothyroid
spinal cord lesons
cows milk intolerance

sexual abuse

How well did you know this?
1
Not at all
2
3
4
5
Perfectly
138
Q

How should constipation be investigated?

A

examination
look for red flags
FBC, TFTs, anti TTG
long term/suspicion of hirschsprung’s > paeds referral

How well did you know this?
1
Not at all
2
3
4
5
Perfectly
139
Q

How should constipation be tx?

A

reassurance, hydration, toilet habits (scheduling visits, bowel diary, rewards, high fibre diet

osmotic laxative eg movicol = 1st line
+/- stimulant laxative eg senna
may require disimpaction regimen initially
tx for 6m - slowly wean off

How well did you know this?
1
Not at all
2
3
4
5
Perfectly
140
Q

What is bronchiolotis?

A

viral LFTI leading to inflammation and infection of the bronchioles

How well did you know this?
1
Not at all
2
3
4
5
Perfectly
141
Q

What is the most common cause of bronchiolitis?

A

RSV - respiratory syncytial virus

How well did you know this?
1
Not at all
2
3
4
5
Perfectly
142
Q

Which children is bronchiolitis more likely to happen to?

A

very common in children under 1yr (mostly under 6mo) in winter

How well did you know this?
1
Not at all
2
3
4
5
Perfectly
143
Q

Why does bronchiolitis affect children differently to adults?

A

smaller airways > small amount of inflammation/mucus in airway has significant effect on infant’s ability to circulate air to the alveoli > harsh breath sounds

How well did you know this?
1
Not at all
2
3
4
5
Perfectly
144
Q

What are the S&S of bronchiolitis?

A

coryzal sx: runny/snotty nose, sneezing, mucus in throat, watery eyes
precede a dry cough

signs of resp distress
dyspnoea, tachypnoea
poor feeding
mild fever
apnoeas
wheeze + crackles on auscultation

How well did you know this?
1
Not at all
2
3
4
5
Perfectly
145
Q

What are the signs of resp distress?

A

raised resp rate
use of accessory muscles to breath - sternocleidomastoid, abdo, intercostal
inter/subcostal recessions
nasal flaring
head bobbing
tracheal tugging
cyanosis
abnormal airway noises

How well did you know this?
1
Not at all
2
3
4
5
Perfectly
146
Q

What is a wheeze? When is it heard? Where from?

A

continuous whistling musical sound
polyphonic - caused by air being forced through multiple lower airways
affects lower airways, comes from lungs
heard more on expiration

How well did you know this?
1
Not at all
2
3
4
5
Perfectly
147
Q

What is stridor? When is it heard? Where from?

A

high pitched turbulent sound, monophonic - through singular upper airway
affects upper airway outside of chest cavity
heard more in inspiration

= emergency - sign of impending airway obstruction

How well did you know this?
1
Not at all
2
3
4
5
Perfectly
148
Q

What are the causes of a wheeze?

A

asthma, COPD
bronchiolitis, bronchitis, bronchiectasis
viral induced wheeze
pneumonia
pulmonary oedema
PE
HF

How well did you know this?
1
Not at all
2
3
4
5
Perfectly
149
Q

What are the causes of stridor?

A

croup
epiglottitis
bacterial tracheitis
diphtheria
laryngomalacia
inhaled foreign body
angioedema/anaphylaxis

How well did you know this?
1
Not at all
2
3
4
5
Perfectly
150
Q

Why does grunting occur?

A

exhaling with glottis partially closed to increased +ve end-exp pressure

How well did you know this?
1
Not at all
2
3
4
5
Perfectly
151
Q

How long does bronchiolitis tend to last?

A

starts as an URTI with coryzal sx - usually half get better

other half develop chest sx
sx worst on day 3/4
last 7-10 days total and recover within 2/3 weeks

How well did you know this?
1
Not at all
2
3
4
5
Perfectly
152
Q

Infants who have had bronchiolitis are at higher risk of what in childhood?

A

viral induced wheeze

How well did you know this?
1
Not at all
2
3
4
5
Perfectly
153
Q

How is bronchiolitis managed?

A

usually supportive mx - adequate intake, saline nasal drops
humidified o2

admission if <3mo/pre-existing condition/clinically unwell
supplementary o2 if sats below 92%
ventilatory support if required

How well did you know this?
1
Not at all
2
3
4
5
Perfectly
154
Q

What are the 3 stages of ventilatory support?

A
  1. high-flow humidified O2: via tight nasal cannula, delivers air and O2 continuously with some added pressure, adds +ve end-exp pressure (PEEP) to maintain airway at end of exp
  2. continuous positive airway pressure (CPAP): sealed nasal cannula that can deliver much higher and more controlled pressures
  3. intubation + ventilation: inserting endotracheal tube into trachea to full control ventilation
How well did you know this?
1
Not at all
2
3
4
5
Perfectly
155
Q

What is the best test to monitor resp distress/children on ventilatory support? What results indicate poor ventilation?

A

capillary blood gas

rising pCO2 - airways have collapsed and can’t clear CO2
falling pH - CO2 building up and not able to buffer acidosis this creates = resp acidosis, if + hypoxic = T2RF

How well did you know this?
1
Not at all
2
3
4
5
Perfectly
156
Q

What preventative monthly injection can be given for bronchiolitis caused by RSV? Which babies are given it?

A

IM palivizumab (mab) - provides passive protection
for high risk babies eg premature, congenital heart disease, CF, bronchopulmonary dysplasia

How well did you know this?
1
Not at all
2
3
4
5
Perfectly
157
Q

How is bronchiolitis diagnosed? What is a DDx that must be ruled out?

A

clinically

CXR to rule out pneumonia - suspect if high fever + localised signs suggesting consolidation

How well did you know this?
1
Not at all
2
3
4
5
Perfectly
158
Q

What is croup?

A

URTI causing oedema in the larynx, trachea and bronchi (acute laryngotracheobronchitis)

How well did you know this?
1
Not at all
2
3
4
5
Perfectly
159
Q

What is croup caused by?

A

parainfluenza virus

sometimes influenza, adenovirus, RSV

How well did you know this?
1
Not at all
2
3
4
5
Perfectly
160
Q

When/which children get croup?

A

children between 6mo and 6yrs, in autumn

How well did you know this?
1
Not at all
2
3
4
5
Perfectly
161
Q

What are the S&S of croup?

A

initially low grade fever + coryzal sx

stridor
hoarse voice
BARKING/SEAL-LIKE COUGH in cluster episodes
breathing difficulty
nocturnal sx

How well did you know this?
1
Not at all
2
3
4
5
Perfectly
162
Q

How is croup treated?

A

supportive tx at home usually - reassurance, fluids, rest, measures taken to avoid spread, if <12mo - low threshold for admission

1st line = oral dexamethasone (2nd = prednisolone)
usually improves in 48hrs, repeat after 12hrs if not better

then stepwise:
O2
nebulised budesonide
nebulised adrenaline if severe
intubation + ventilation

How well did you know this?
1
Not at all
2
3
4
5
Perfectly
163
Q

Croup caused by diptheria causes what?

A

epiglottitis - high mortality

How well did you know this?
1
Not at all
2
3
4
5
Perfectly
164
Q

What is epiglottitis caused by?

A

haemophilus influenza B

others - strep pyogenes, strep pneumoniae

How well did you know this?
1
Not at all
2
3
4
5
Perfectly
165
Q

Why has the incidence of influenza reduced?

A

HiB vaccine - be suspicious in children without the vaccine

How well did you know this?
1
Not at all
2
3
4
5
Perfectly
166
Q

What are the S&S of epiglottitis?

A

very acute onset
4Ds: dysphagia, dysphonia (muffled voice), distress, DROOLING
extremely sore throat
high fever, look septic/unwell
no/minimal cough
tripod position - sx improve on sitting upright and learning forward (often child will be immobile sitting upright with open mouth)

increasing resp difficulty
inspiratory SOFT STRIDOR

How well did you know this?
1
Not at all
2
3
4
5
Perfectly
167
Q

How is epiglottitis investigated?

A

no time!!

in theory:
laryngoscopy - beefy-red-stuff oedamatous epiglottitis = diagnostic
lateral neck XR - thumbprint sign
exclude foreign body

How well did you know this?
1
Not at all
2
3
4
5
Perfectly
168
Q

How is epiglottitis managed?

A

life threatening emergency - need senior paediatrician + anaesthetist
DO NOT EXAMINE THROAT + don’t panic pt

protect airway - nasotracheal intubation if needed, most don’t need
emergency tracheostomy if throat closes - admit to ITU if intubated

BCs once airway secured
IV cefotriaxone + dexamethasone

rifampicin for close contact prophylaxis

How well did you know this?
1
Not at all
2
3
4
5
Perfectly
169
Q

What are the complications of epiglottitis?

A

death
epiglottic abscess - collection of pus around epiglottitis, also a life-threatening emergency, tx similar

How well did you know this?
1
Not at all
2
3
4
5
Perfectly
170
Q

How is croup and epiglottitis differentiated?

A

epiglottitis can present similar but with a more rapid onset
look for: unvaccinated children, fever, sore throat, difficulty swallowing, sitting forward and drooling

How well did you know this?
1
Not at all
2
3
4
5
Perfectly
171
Q

How is croup differentiated from bacterial tracheitis?

A

BT has more toxic appearance

How well did you know this?
1
Not at all
2
3
4
5
Perfectly
172
Q

How is croup differentiated from laryngomalacia?

A

laryngomalacia has no chest signs

How well did you know this?
1
Not at all
2
3
4
5
Perfectly
173
Q

What conditions are asthma a/w?

A

atopic conditions eg eczema, hay fever, food allergies
fhx of atopic conditions

How well did you know this?
1
Not at all
2
3
4
5
Perfectly
174
Q

What triggers are asthma a/w?

A

infection
exercise
cold air
dust
animals
smoke/pollution
food allergens
NSAIDs/beta blockers

How well did you know this?
1
Not at all
2
3
4
5
Perfectly
175
Q

What are the 3 components in the pathophysiology of asthma?

A

reversible increase in airway resistance in response to irritant stimuli
caused by:
1. bronchial smooth muscle contraction - hypersensitive
2. mucosal inflammation (t-helper cell activation + cytokine production)
3. increased mucus production

How well did you know this?
1
Not at all
2
3
4
5
Perfectly
176
Q

What is bronchoconstriction caused by asthma reversible with?

A

bronchodilators

How well did you know this?
1
Not at all
2
3
4
5
Perfectly
177
Q

How does the sympathetic and parasympathetic nervous system affect bronchial calibre?

A

sympathetic: bronchodilation + decreases mucous secretion via B2-adrenoceptors

parasympathetic: bronchoconstriction + increases mucous secretion via M3 receptors

How well did you know this?
1
Not at all
2
3
4
5
Perfectly
178
Q

What features of a history make a resp presentation less likely to be asthma?

A

wheeze only related to coughs/colds > viral induced wheeze
isolated/productive cough
normal ix
no response to treatment
unilateral wheeze > focal lesion/foreign body/infection

How well did you know this?
1
Not at all
2
3
4
5
Perfectly
179
Q

What are the sx of asthma?

A

dry cough with bilateral widespread polyphonic wheeze, SoB and a tight chest
diurnal variability - worse at night and early morning
episodic sx with intermittent exacerbations
sx improve with bronchodilator

typical triggers
low exercise tolerance
hx/fhx of other atopic conditions
sx better on days off work

How well did you know this?
1
Not at all
2
3
4
5
Perfectly
180
Q

What are the signs of an acute asthma attack?

A

progressively worsening SoB
signs of resp distress
tachypnoea

How well did you know this?
1
Not at all
2
3
4
5
Perfectly
181
Q

What obs and signs indicate a severe/life-threatening asthma attack?

A

SpO2 < 92%

severe = too breathless to talk/feed, HR >140, RR >40, use of accessory muscles
life-threatening = silent chest, poor resp effort, agitation, altered consciousness, cyanosis, trachea tug, subcostal recessions

How well did you know this?
1
Not at all
2
3
4
5
Perfectly
182
Q

What kind of wheeze is a/w asthma?

A

Bilateral widespread expiratory polyphonic wheeze

How well did you know this?
1
Not at all
2
3
4
5
Perfectly
183
Q

What chest sounds are a red flag in an asthmatic child?

A

silent chest > chest so tight that child can’t move air through airways to create a wheeze
a/w reduced resp effort due to fatigue
no wheeze doesn’t always = no resp distress!

How well did you know this?
1
Not at all
2
3
4
5
Perfectly
184
Q

How is asthma diagnosed?

A

diagnosis = ix demonstrate variable airway obstruction/inflammation + clinical picture
if ix inconclusive > try tx and monitor sx - improvement = diagnostic

5-16yrs:
1. spirometry
2. if +ve > BDR test
3. still uncertain > fractional exhaled NO
4. still uncertain > monitor PEF variability for 2-4 weeks

under 5yrs: based on S&S, review regularly, do ix if still symptomatic at 5yrs

How well did you know this?
1
Not at all
2
3
4
5
Perfectly
185
Q

What are the 2 mainstays of asthma treatment?

A

bronchodilator relievers
anti-inflammatory preventors

How well did you know this?
1
Not at all
2
3
4
5
Perfectly
186
Q

How is asthma treated in <5yrs?

A
  1. SABA inhaler PRN
  2. add low dose corticosteroid inhaler OR leukotriene antagonist
  3. add other option from step 2
  4. refer
How well did you know this?
1
Not at all
2
3
4
5
Perfectly
187
Q

How is asthma treated in >5yrs?

A
  1. SABA PRN
  2. add low dose corticosteroid inhaler
  3. add LABA, continue only if good response
  4. titrate up corticosteroid to medium dose, consider adding an oral leukotriene receptor antagonist/oral theophylline/inhaled LAMA
  5. titrate up corticosteroid to high dose, combine additional tx from step 4/add oral beta-2 agonist
  6. refer
  7. consider oral daily steroids at lowest possible dose
How well did you know this?
1
Not at all
2
3
4
5
Perfectly
188
Q

What are the basic steps of treating an acute asthma attack?

A

O2 (if sats <94%/working hard)
bronchodilators - beta agonist +++
steroids - PO prednisolone/IV hydrocortisone (reduce airway inflammation)
abx if bacterial cause suspected - amoxicillin/erythromycin

How well did you know this?
1
Not at all
2
3
4
5
Perfectly
189
Q

How should a salbutamol inhaler be used for a mild asthma attack?

A

regular salbutamol inhalers via a spacer
4-6 puffs every 4hrs

How well did you know this?
1
Not at all
2
3
4
5
Perfectly
190
Q

How should a mod-severe asthma attack be managed?

A

stepwise until control achieved:
1. salbutamol inhalers via spacer - start with 10 puffs every 2hrs
2. nebulisers with salbutamol/ipratropium bromide
3. oral prednisolone - 1mg/kg 1/day for 3 days
4. IV hydrocortisone
5. IV MgSO4
6. IV salbutamol
7. IV aminophylline
peak flow before and after tx, must be >75% after tx before discharge
control achieved > gradually work back down

still no control > need anaesthetist + ICU, consider intubation + ventilation

How well did you know this?
1
Not at all
2
3
4
5
Perfectly
191
Q

A child can be discharged after an acute asthma attack when they are needing how much salbutamol?

A

well on 6 puffs 4hrly
prescribe reducing regime for home to get down to 2-4 puffs PRN

How well did you know this?
1
Not at all
2
3
4
5
Perfectly
192
Q

What is a possible dangerous SE of salbutamol?

A

causes K+ to be absorbed into cells > hypokalaemia
consider monitoring serum K+

also tachycardia, tremor

How well did you know this?
1
Not at all
2
3
4
5
Perfectly
193
Q

What is the most common reason for uncontrolled asthma?

A

poor technique

How well did you know this?
1
Not at all
2
3
4
5
Perfectly
194
Q

What medications are CI in asthma?

A

NSAIDS - inhibit COX pathway > promote arachidonic acid conversion to leukotrienes > can provoke asthma

Beta-blockers > CI!!! - reduce effectiveness of B2-agonists

How well did you know this?
1
Not at all
2
3
4
5
Perfectly
195
Q

What is the fractional exhaled nitric oxide test for asthma?

A

measured exhaled NO = marker of eosinophilic inflammation

How well did you know this?
1
Not at all
2
3
4
5
Perfectly
196
Q

What is spirometry? What is a poor result?

A

measures volume of air expelled from lungs after maximal inspiration

FEV1:FVC < 70%

How well did you know this?
1
Not at all
2
3
4
5
Perfectly
197
Q

How can spirometry be used to check reversibility?

A

bronchodilator reversibility: check spirometry change before and 15 mins after SABA inhalation

improvement in FEV1 of 12%+

How well did you know this?
1
Not at all
2
3
4
5
Perfectly
198
Q

What is peak expiratory flow? What is a result indicative of asthma?

A

measures max speed of expiration
monitor twice daily for 2-4wks

variability 20%+

How well did you know this?
1
Not at all
2
3
4
5
Perfectly
199
Q

What is a direct bronchial challenge test?

A

measures change in spirometry after methacholine/histamine inhalation

How well did you know this?
1
Not at all
2
3
4
5
Perfectly
200
Q

How does a SABA work? Give an example? What are the SEs?

A

B2-agonism > bronchial smooth muscle relaxation > dilates bronchi > improves airflow in obstructed airway

eg salbutamol
SEs: TREMOR, tachycardia, palpitations

How well did you know this?
1
Not at all
2
3
4
5
Perfectly
201
Q

How do ICS work? Give an example? What are the SEs?

A

reduce leukocyte proliferation + downregulate pro-inflammatory cytokine, leukotriene and chemokine production > reduces mucosal inflammation, dilates airways + reduces mucous secretion

eg beclometasone
SEs: ORAL CANDIDIASIS, hoarse voice, regular high dose > adrenal suppression (monitor growth)

How well did you know this?
1
Not at all
2
3
4
5
Perfectly
202
Q

How do leukotriene receptor antagonists work? Give an example.

A

down-regulate inflammatory leukotrienes

eg montelukast

How well did you know this?
1
Not at all
2
3
4
5
Perfectly
203
Q

Give an example of a LABA. What must they always be given alongise?

A

salmeterol
given alongside an ICS

How well did you know this?
1
Not at all
2
3
4
5
Perfectly
204
Q

Give an example of a LAMA. How do they work?

A

tiotropium

bind to M3 muscarinic receptors and block bronchoconstriction effect of acetylcholine at these receptors in the airway smooth muscle

How well did you know this?
1
Not at all
2
3
4
5
Perfectly
205
Q

What class of drug is theophylline?

A

xanthine - bronchodilator

How well did you know this?
1
Not at all
2
3
4
5
Perfectly
206
Q

What is viral induced wheeze?

A

acute wheezy illness caused by viral infection

How well did you know this?
1
Not at all
2
3
4
5
Perfectly
207
Q

What viruses generally cause a viral induced wheeze?

A

RSV, rhinovirus

How well did you know this?
1
Not at all
2
3
4
5
Perfectly
208
Q

Why do viral illnesses lead to a viral induced wheeze?

A

children have small airways > inflammation + oedema occurs in virus > restrict air flow + inflammation triggers bronchoconstriction = proportionally large air flow restriction

How well did you know this?
1
Not at all
2
3
4
5
Perfectly
209
Q

How do distinguish between a viral induced wheeze and asthma?

A

VIW: present before 3yrs, no atopic hx, only occurs during viral infections

asthma: can be triggered by a viral infection but has lots of other triggers, variable and reversible airflow obstruciont

How well did you know this?
1
Not at all
2
3
4
5
Perfectly
210
Q

What is the classic presentation of a viral induced wheeze?

A

viral illness (fever, cough, coryzal sx) for 1-2 days before:
SoB
signs of resp distress
expiratory wheeze throughout the chest

How well did you know this?
1
Not at all
2
3
4
5
Perfectly
211
Q

What should a focal wheeze make you concerned about?

A

inhaled foreign body
tumour

How well did you know this?
1
Not at all
2
3
4
5
Perfectly
212
Q

How is viral induced wheeze managed?

A

same as acute asthma attack

How well did you know this?
1
Not at all
2
3
4
5
Perfectly
213
Q

What is the genetic mutation that causes CF? How is it passed on?

A

AR
mutation of the CF transmembrane conductance regulatory gene on chromosome 7 > affects mucous glands
most common variation affects a particular type of Cl- channel

How well did you know this?
1
Not at all
2
3
4
5
Perfectly
214
Q

What are the key consequences of the CF mutation?

A
  1. thick pancreatic + biliary secretions > blockage of the ducts > lack of digestive enzymes in digestive tract eg pancreatic lipase
  2. low volume thick airway secretion > reduce airway clearance > bacterial colonisation > susceptibility to airway infections
  3. congenital bilateral absence of vas deferens in males > healthy sperm but no route from testes to ejaculate > male infertility
How well did you know this?
1
Not at all
2
3
4
5
Perfectly
215
Q

How is CF screened for?

A

newborn bloodspot test

How well did you know this?
1
Not at all
2
3
4
5
Perfectly
216
Q

What is usually the first sign of CF?

A

meconium ileus > meconium thick and sticky > gets stuck and obstructs the bowel
> no meconium passed within 24hrs, abdominal distention, vomiting
= pathognomonic for CF

How well did you know this?
1
Not at all
2
3
4
5
Perfectly
217
Q

What are the sx of CF?

A

chronic cough
thick sputum production
recurrent RTIs
steatorrhoea (lack of lipase)
abdo pain + bloating
salty sweat
FtT - poor weight and height gain

How well did you know this?
1
Not at all
2
3
4
5
Perfectly
218
Q

What are the signs of CF?

A

low weight/heigh on growth chart
nasal polyps
finger clubbing
crackles/wheezes on auscultation
abdo distention

How well did you know this?
1
Not at all
2
3
4
5
Perfectly
219
Q

What are the causes of clubbing in children?

A

hereditary
cardiac: cyanotic heart disease, IE
resp: CF, TB
gastro: IBD, liver cirrhosis

How well did you know this?
1
Not at all
2
3
4
5
Perfectly
220
Q

What are the three key diagnostic tests for CF?

A

newborn blood spot testing - picks up most cases
GS = sweat test
genetic testing for CFTR gene (on amniocentesis, chorionic villous sampling or as blood test after birth)

How well did you know this?
1
Not at all
2
3
4
5
Perfectly
221
Q

What is the CF sweat test?

A

electrodes placed either side of patch of skin > small current passed causes skin to sweat > sweat absorbed and tested for Cl- conc

How well did you know this?
1
Not at all
2
3
4
5
Perfectly
222
Q

What are the most common colonisers in the lungs in children with CF?

A

struggle to clear secretions > lots of moisture and O2 for colonies

staph aureus
pseudomonas

How well did you know this?
1
Not at all
2
3
4
5
Perfectly
223
Q

What can CF pts take long-term to prevent staph aureus chest infection?

A

prophylactic flucloxacillin

How well did you know this?
1
Not at all
2
3
4
5
Perfectly
224
Q

Why are children with CF advised to avoid contact with each other?

A

pseudomonas difficult to get rid of - becomes resistant to multiple abx
significant increase in morbidity and mortality
easily passed on between CF pts

How well did you know this?
1
Not at all
2
3
4
5
Perfectly
225
Q

How is a pseudomonas colonisation treated in a child with CF?

A

long-term nebulised abx eg tobramycin

or oral ciprofloxacin

How well did you know this?
1
Not at all
2
3
4
5
Perfectly
226
Q

How is CF managed?

A

chest physio + exercise
prophylactic fluclox
bronchodilators
nebulised DNase - breaks down DNA material in resp secretions > less viscous
nebulised hypertonic saline

high calorie diet - due to malabsorption, increased resp effort, coughing, infections
CREON (pancrelipase)

vaccines - pneumococcal, influenza, varicella

How well did you know this?
1
Not at all
2
3
4
5
Perfectly
227
Q

What conditions do CF pts need to be regularly monitored and screened for?

A

pseudomonas - sputum
diabetes
OP, vit D deficiency
liver failure

How well did you know this?
1
Not at all
2
3
4
5
Perfectly
228
Q

What is the prognosis of CF? What affects the prognosis?

A

depends on severity of sx, type of mutation, adherence to tx, frequency of infection/lifestyle

median LE = 47yrs

90% have pancreatic insufficiency
50% have CF related diabetes
30% have liver disease
most males infertile

How well did you know this?
1
Not at all
2
3
4
5
Perfectly
229
Q

What is laryngomalacia?

A

the supraglottic larynx is structured in a way that causes partial airway obstruction

short aryepiglottic folds > pulls on epiglottis = omega shape > tissue surrounding is softer/less tone > can flop across the airway, esp during inspiration = partial occlusion

How well did you know this?
1
Not at all
2
3
4
5
Perfectly
230
Q

How does laryngomalacia present?

A

intermittent inspiratory stridor, prominent on feeding, upset, lying on back, in URTI
no resp distress

generally resolves with age

How well did you know this?
1
Not at all
2
3
4
5
Perfectly
231
Q

How does pneumonia present?

A

cough - wet, productive
high fever
tachypnoea, tachycardia
increased work of breathing
lethargy
delirium

How well did you know this?
1
Not at all
2
3
4
5
Perfectly
232
Q

What are the characteristic chest sounds heard in pneumonia?

A

bronchial breath sounds - harsh sounds equally loud in insp + exp due to consolidation of lung tissue around airway

focal coarse crackles - due to air passing through sputum

dullness to percussion - due to lung collapse or consolidation

How well did you know this?
1
Not at all
2
3
4
5
Perfectly
233
Q

What is the most common bacterial and viral cause of pneumonia?

A

bact: strep pneumonia

viral: RSV

How well did you know this?
1
Not at all
2
3
4
5
Perfectly
234
Q

What is the 1st line mx for pneumonia?

A

amoxicillin

+ macrolide eg erythromycin to cover atypical pneumonia/pencillin allergy
IV if sepsis/malabsorption

How well did you know this?
1
Not at all
2
3
4
5
Perfectly
235
Q

What virus is mumps caused by?

A

RNA paramyxovirus - mumps

How well did you know this?
1
Not at all
2
3
4
5
Perfectly
236
Q

How is mumps transmitted?

A

resp droplets

How well did you know this?
1
Not at all
2
3
4
5
Perfectly
237
Q

How does mumps present?

A

prodrome of coryzal sx - fever, muscle aches, lethargy, reduced appetite, headache

PAROTID GLAND SWELLING - uni/bilateral, a/w pain
trismus - spasm of muscles of mastication when chewing
possibly ear ache

How well did you know this?
1
Not at all
2
3
4
5
Perfectly
238
Q

What are the possible complications of mumps?

A

pancreatitis (abdo pain)
orchitis (testicular pain + swelling)
meningitis/encephalitis (confusion, neck stiffness, headache)
sensorineural hearing loss

How well did you know this?
1
Not at all
2
3
4
5
Perfectly
239
Q

How is mumps managed?

A

notifiable disease

supportive - rest, fluids, analgesia
school exclusion for 7 days

How well did you know this?
1
Not at all
2
3
4
5
Perfectly
240
Q

What virus causes measles?

A

RNA paramyxovirus - morbillivirus

How well did you know this?
1
Not at all
2
3
4
5
Perfectly
241
Q

How does measles present?

A

catarrhal stage 10-12 days after exposure: 4Cs - cough, cranky, coryza, conjunctivitis

koplik spots 2 days after fever - grey/white spots on buccal mucosa = pathognomonic

macular erythematous rash 3-5 days after fever starts - starts on face, classically behind ears, spreads down

How well did you know this?
1
Not at all
2
3
4
5
Perfectly
242
Q

What are the possible complications of measles?

A

1/3rd have a complication

pneumonia
diarrhoea, dehydration
encephalitis, meningitis
hearing/vision loss

How well did you know this?
1
Not at all
2
3
4
5
Perfectly
243
Q

What is the most common complication of measles?

A

otitis media

How well did you know this?
1
Not at all
2
3
4
5
Perfectly
244
Q

How is measles managed?

A

rest
isolation for 4 days from rash onset

notifiable disease
highly contagious

How well did you know this?
1
Not at all
2
3
4
5
Perfectly
245
Q

What virus causes rubella?

A

RNA paramyxovirus - rubivirus

How well did you know this?
1
Not at all
2
3
4
5
Perfectly
246
Q

What are the clinical features of rubella?

A

coryzal prodrome - mild fever, sore throat

milder pink macular erythematous rash than measles, starts on face and spreads to rest of body, lasts about 3 days
joint pain
lymphadenopathy - sub occipital/posterior auricular

How well did you know this?
1
Not at all
2
3
4
5
Perfectly
247
Q

How are the MMR diseases diagnosed?

A

clinically

saliva swab for igM as all are notifiable

How well did you know this?
1
Not at all
2
3
4
5
Perfectly
248
Q

How is rubella managed?

A

isolation for 5 days after rash appears
rest

How well did you know this?
1
Not at all
2
3
4
5
Perfectly
249
Q

Which group of people is rubella most dangerous in?

A

pregnant women

<13 wks transmission to fetus = 80% > TOP offered
>16 wks transmission = 25%, unlikely to cause defects

How well did you know this?
1
Not at all
2
3
4
5
Perfectly
250
Q

What is the congenital rubella triad?

A

sensorineural deafness
cardiac abnormalities
eye abnormalities eg cataracts

How well did you know this?
1
Not at all
2
3
4
5
Perfectly
251
Q

How does parvovirus B19/fifth disease/slapped cheek syndrome present?

A

coryzal prodrome, fever, muscle aches, lethargy

2-5 days later: sudden diffuse malar rash on cheeks
few days later: reticular (net-like) mildy erythematous rash on trunk and limbs, can be raised and itchy
= fade after 1-2wks
arthropathy

252
Q

What are the possible complications of parvovirus B19?

A

at risk people: pregnant, immunocompromised, haematological conditions (thalassaemia, sickle cell, HS, haemolytic anaemia)

aplastic anaemia! (check FBC + reticulocytes in at risk people)
encephalitis/meningitis

pregnancy complications incl death, hydrops fetalis

253
Q

What virus causes hand, foot and mouth disease?

A

coxsackie A

254
Q

How does hand, foot and mouth disease present?

A

initially coryzal sx eg temp, cough, fatigue

after 1-2 days: small mouth + tongue ulcers, may be painful
followed by blistering red spots across body - most on hands, feet and around mouth, may be itchy

255
Q

How is hand, foot and mouth disease managed?

A

rest, fluid, analgesia

highly contagious - avoid sharing towels/bedding etc
no school exclusion

256
Q

Are most cases tonsilitis viral or bacterial?

A

viral

257
Q

What is the most common bacterial causes of tonsilitis?

A

group A beta haemolytic strep - strep pyogenes

258
Q

What is the centor criteria?

A

probability that tonsilitis is due to bacteria

score 3+: 40-60% probability of bacterial tonsilitis = offer abx
- fever >38
- tonsillar exudates
- absence of cough
- tender anterior cervical lymphadenopathy

259
Q

What is the FeverPAIN score?

A

alternative to centor criteria
2-3 = 34-40% prob
4-5 = 62-65% prob

Fever during previous 24hrs
Purulence (pus on tonsils)
Attended within 3 days of onset of sx
Inflamed tonsils
No cough or coryza

260
Q

When should abx be given to a pt with tonsilitis?

A

centor 3+ or feverpain 4+
young infant/immunocompromised/comorbidities/very unwell
hx of rheumatic fever

if feverpain 2-3: consider delayed abx and advise to use if sx don’t improve in 3-5 days/if sx worsen

261
Q

What is 1st line tx for bacterial tonsilitis?

A

pencillin V (phenoxymethylpenicillin) for 10 days

clarithromycin in penicillin allergy

DON’T give amox in case it is caused by infectious mononucleosis > causes rash

262
Q

What are the complications of tonsilitis?

A

chronic tonsilitis
peritonsillar abscess (quinsy)
otitis media
scarlet or rheumatic fever

263
Q

What is the cause of scarlet fever?

A

endotoxins released by group a beta haemolytic strep (strep pyogenes)

264
Q

What are the clinical features of scarlet fever?

A

12-48hrs after sore throat onset: red/pink blotchy macular rash that feels like sandpaper, starts on the trunk and spreads out, red flushed cheeks, circumoral pallor

strawberry tongue (can be white also)
cervical lymphadenopathy

265
Q

How is scarlet fever tx?

A

same as tonsilitis - phenoxymethylpencillin

266
Q

What virus causes chickenpox?

A

varicella zoster
once they have an episode = develop immunity to VZV

267
Q

How is the chickenpox rash described?

A

widespread erythematous raised vesicular (fluid filled) blistering lesions
starts on trunk/face and spreads outwards affecting whole body
lesions scab over eventually = stop being contagious

268
Q

How is chickenpox transmitted?

A

highly contagious - stops being contagious when lesions scab over
direct contact with lesions/infected droplets from cough or sneeze

269
Q

Why can chickenpox lead to shingles?

A

virus can lie dormant in sensory dorsal root gangion cells
CNs reactivate later in life as shingles or ramsay hunt syndrome

270
Q

What happens to the fetus with chickenpox during pregnancy?

A

<28wks > developmental problems in a small number of fetuses (congenital varicella syndrome)

271
Q

Why can chickenpox be worse during pregnancy?

A

more severe cases eg varicella pneumonitis, hepatitis, encephalitis

check IgG for VZV, offer vaccine

272
Q

What happens to the neonate if the mother has chickenpox during delivery?

A

risk of life threatening neonatal infection - needs VZ Igs + aciclovir

273
Q

What effects does congenital varicella syndrome have on the fetus?

A

when infection occurs <28wks:
FGR
microcephaly, hydrocephalus, LD
scars/sig skin changes in specific dermatomes
limb hypoplasia
cataracts, chorioretinitis

274
Q

How is chickenpox managed?

A

usually mild and self limiting

acyclovir considered in immunocompromised/neonates
itching - calamine lotion/antihistamine
signs of infection - flucloxacillin

exclusion from school/avoid pregnant women until lesions are dry/crusted over - usually 5 days after rash onset

275
Q

What are the possible complication of chickenpox?

A

bacterial superinfection
pneumonia
encephalitis
dehydration
conjunctival lesions

shingles/ramsey hunt later in life

276
Q

What bacteria caused impetigo?

A

staph aureus

277
Q

How does impetigo present?

A

pustules + blisters on an erythematous based
blisters leave brown GOLDEN CRUST when they burst - often around mouth

may be feverish/unwell

278
Q

What is impetigo? What children does it happen in?

A

highly contagious superficial skin infection
peaks ages 2-5yrs
enters through break in skin - health skin or a/w eczema/dermatitis

279
Q

How is impetigo managed?

A

1st line = topical hydrogen peroxide 1% or topical fusidic acid
severe/widespread > oral flucloxacillin

school exclusion until lesions have crusted over/48hrs+ after abx started

280
Q

What are the causes of otitis externa?

A

bacterial - mostly pseudomonas aeruginosa /staph aureus

others: fungal (think if had multiple courses of topical abx), eczema, dermatitis

281
Q

What are the RFs for otitis externa?

A

humidity
swimming
scratching from eczema/psoriasis
trauma to ear

ear wax = protective

282
Q

What are the clinical features of otitis externa?

A

pain - pulling at ear
scaly skin
discharge
conductive hearing loss on affected side

283
Q

What needs to be excluded before giving aminoglycosides to a pt with otitis externa?

A

eg neomycin, gentamicin

= ototoxic > can cause hearing loss if gets past TM > ensure TM is intact

284
Q

What is a severe life-threatening complication of otitis externa?

A

malignant OE

progresses to bones surrounding ear > osteomyelitis of the temporal bone
usually linked to susceptibility to infection

headache, severe pain, fever
key finding = granulation tissue

> admission, IV abx, imaging

285
Q

How is otitis externa treated?

A

mild: acetic acid 2% drops (anti fungal/bacterial)

moderate: topical abx + steroid eg neomycin, dexamethasone and acetic acid

severe/systemic sx: oral abx eg fluclox/discuss with ENT

Ear wick can be used if spray/drops difficult

if fungal: clotrimazole drops

286
Q

What is the most common cause of otitis media?

A

strep pneumoniae

often preceded by viral URTI

287
Q

How does otitis media present?

A

EAR PAIN
reduced hearing
coryzal sx/URTI signs
generally unwell

effect on vestibular system > balance issues, vertigo
discharge = RF for tympanic membrane rupture

288
Q

How will a tympanic membrane appear normally and in otitis media?

A

normal: pearly grey translucent slightly shiny, malleus visible, cone of light reflecting light of otoscope

otitis media: bulging red inflamed membrane, discharge/hole in tympanic membrane if perforated

289
Q

How is otitis media managed?

A

most resolve without abx

indications for immediate abx:
children <2yrs with bilateral otitis media
otorrhoea/perforated TM
in only hearing ear/cochlear implant
immunocompromised

could give delayed + safety net

give if >4 days sx

1st line = amoxicillin for 5-7 days (otherwise erythro/clarithromycin)

290
Q

What are the potential complications of otitis media?

A

mastoiditis
TM perforation
intracranial abscess

291
Q

What is glue ear/otitis media with effusion? WHy does it happen?

A

middle ear inflammation with collection of fluid behind TM

eustachian tube drains secretions from middle ear > if blocked fluid builds up there

292
Q

What is the key sx of glue ear?

A

reduction in hearing

risk of otitis media

293
Q

How does glue ear appear on otoscopy?

A

grey dull tympanic membrane with air bubbles/visible fluid level + loss of cone of light reflex

294
Q

How is glue ear managed?

A

audiometry referral

usually conservative tx - most resolve without tx in 3mo
otovent - symptomatic relief to help open/clear eustachian tube
consider grommets/hearing aids if >3mo/co-morbidities

295
Q

What are the possible causes/RFs for glue ear?

A

eustachian tube dysfunction - shorter/more horizontal tubes
URTI > adenoid hypertrophy which blocks eustachian tube
increased risk: Down’s, cleft palate

296
Q

What is transient synovitis?

A

aka irritable hip

transient irritation and inflammation in the synovial membrane of the joint

297
Q

What is the most common cause of hip pain in 3-10yr olds?

A

transient synovitis

298
Q

How does transient synovitis present?

A

recent viral URTI

acute or gradual onset of:
- limp
- refusal to weight bear
- groin/hip pain - mostly on movement, may improve throughout the day
- mild low grade temp
otherwise well - normal signs, no systemic illness

299
Q

Joint pain + fever is a red flag for what?

A

septic arthritis

300
Q

How is transient synovitis managed?

A

analgesia
safety net advice if they develop a fever
follow up

301
Q

What is the prognosis of transient synovitis?

A

sig improvement after 48hrs
should resolve within 1-2 wks with no lasting problems
may recur

302
Q

What is perthe’s disease?

A

disruption of blood flow to femoral head > avascular necrosis of the bone affecting the epiphysis

303
Q

Which children does Perthe’s disease occur in?

A

4-12ys, mostly between 5 and 8yrs
more common in boys
idiopathic

304
Q

How does Perthe’s resolve? What are the consequences of this?

A

re/neovascularisation and healing in the femoral head over time > remodelling of bone

leads to a soft deformed femoral head > early hip OA > may need artificial hip replacement

305
Q

How does perthe’s present?

A

slow onset of:
- pain in hip/groin
- limp
- restricted hip movement
- possibly referred pain to knee
- late sign = leg length discrepancy

no hx of trauma

306
Q

How is perthe’s investigated?

A

XR - can be normal

joint space widening/irregularity
crescent sign = late

307
Q

How is perthe’s managed?

A

bed rest, traction, crutches, analgesia - to maintain healthy position/alignment + reduce risk of deformity

physio
regular XR to assess healing
surgery in severe cases/older children/not healing

308
Q

What is SUFE/SCFE?

A

slipped upper/capital femoral epiphysis

head of the femur is displaced along the growth plate

309
Q

Which children do SCFE happen in?

A

more common in boys + obese children
8-15yrs

310
Q

How does SCFE present?

A

typical: adolescent obese male undergoing a growth spurt
possible hx of minor trauma - suspect if pain is disproportionate to severity of trauma

hip/groin/thigh/knee pain
restricted range of hip movement
painful limp

311
Q

On examination, how does SCFE present?

A

pt prefers to keep hip in external rotation - +ve Drehmann’s sign
limited movement of hip
restricted internal rotation

312
Q

How is SCFE investigated?

A

XR = diagnostic

313
Q

How is SCFE managed?

A

surgery - return the femoral head to correct position + fix in place

314
Q

Which children is septic arthritis most common in?

A

<4yrs
joint replacement

315
Q

How does septic arthritis present?

A

affects a single join - knee/hip usually

rapid onset of:
- hot red swollen painful joint
- refusing to weight bear
- stiffness/reduced range of motion, pseuoparesis
- systemic sx - fever, lethargy

can be subtle in young children
look for scratches/infected chickenpox

316
Q

What is the most common causative organism of septic arthritis?

A

staph aureus

317
Q

What are the DDx of a child presenting with joint pain?

A

transient synovitis - no systemic illness, pain mostly on movement, after illness, quick improvement

perthe’s disease - boys, limp, no hx of trauma

SCFE - minor trauma hx, obese adolescent male, restricted movement - likes hip to be in external rotation

JIA -

318
Q

How should septic arthritis be investigated?

A

low threshold for tx - esp in immunocompromised
+ admission

aspirate joint prior to abx if possible
blood cultures, CRP

319
Q

How should septic arthritis be managed?

A

empirical IV abx until sensitivities are known

abx continued for 3-6wks after confirmation
may require surgical drainage + washout of the joint (debridement)

320
Q

What is osteomyelitis?

A

infection in bone/bone marrow
usually in metaphysis of long bones

321
Q

What is the most common causative organism of osteomyelitis?

A

staph aureus

322
Q

How does acute versus chronic osteomyelitis present?

A

acute - rapid onset, acutely unwell child
chronic - deep seated slow growing infection with slowly developing sx

323
Q

What are the possible routes of infection in osteomyelitis?

A

directly into the bone eg open fracture
through the blood
through another route eg skin, gums

324
Q

What are the RFs for osteomyelitis?

A

boys, < 10yrs

open fracture
orthopaedic surgery
immunocompromised, sickle cell, HIV, TB

325
Q

How does osteomyelitis present?

A

refusing to use limb/weight bear
pain
swelling
tenderness

afebrile/low grade fever
acute may have high fever - esp if it spreads to the joints = septic arthritis

326
Q

How is osteomyelitis investigated?

A

XR - can normal
best = MRI

bloods - raised CRP/ESR/WBCs
blood cultures
bone marrow aspiration/bone biopsy with histology + culture

327
Q

How is osteomyelitis managed?

A

prolonged abx therapy
may need surgery for drainage + debridement

328
Q

What is Osgood-Schlatter disease?

A

inflammation at the tibial tuberosity where the patella ligament inserts

329
Q

Which children does Osgood-Schlatter disease occur in?

A

males aged 10-15yrs
sporty boys

330
Q

First differential: anterior knee pain in a male adolescent

A

osgood-schlatter

331
Q

What is the pathophysiology of Osgood-Schlatter disease?

A

stress from running/jumping + growth in epiphyseal plate = inflammation on tibial epiphyseal plate
+
multiple avulsion fractures where ligament pulls away tiny pieces of bone
= growth of tibial tuberosity > lump below the knee

332
Q

How does Osgood-Schlatter disease present?

A

gradual onset:
visible/palpable hard and tender lump at tibial tuberosity (initially tender > becomes hard and non-tender)
pain in anterior aspect of knee exacerbated by physical activity/kneeling/extension of knee

333
Q

How is Osgood-Schlatter disease managed?

A

reduction in physical activity
ice
NSAIDS

stretching/physio once sx settle (usually left with hard boney lump on knee)

334
Q

When is DDH likely to be identified?

A

NIPE

or later with hip asymmetry + reduced range of movement

335
Q

What sx may an adult have with DDH that has persisted?

A

weakness
recurrent subluxation/dislocation
abnormal gait
early degenerative changes

336
Q

What are the RFs for DDH?

A

1st degree FHx
breech
multiple pregnancy

337
Q

What are signs in a NIPE that a baby has DDH?

A

different leg lengths
restricted hip abduction
sig bilateral restriction in abduction
difference in knee level when hips are flexed
clunking of hips on special test

338
Q

What are the 2 special tests for DDH?

A

ortolani: baby on back with hip + knees flexed > palms on baby’s knees with thumbs on inner thigh and fingers on outer thigh > gentle pressure to abduct hips + apply pressure behind legs to see if hips dislocate anteriorly

barlow: baby on back with hips abducted and hip and knees flexed at 90 degrees > gentle downward pressure on knees through femur to see if femoral head dislocates posteriorly

339
Q

What are positive findings for DDH in the 2 special tests? versus a common finding?

A

clicking common
clunking indicates DDH = needs US

340
Q

How is DDH diagnosed?

A

US

XR can be helpful, esp in older children

341
Q

How is DDH managed?

A

<6mo: Pavlik harness - fitted and kept on for about 2mo, hold femoral head in correct position to allow hip socket to establish a normal shape

harness failed/>6mo: surgery, then hip spica cast to immobilise hip

342
Q

What is JIA?

A

AI inflammation of the joints in children

343
Q

What are the 3 requirements for a diagnosis of JIA? And the 3 key features?

A

arthritis without any other cause
lasts >6wks
in a pt <16yrs

joint pain, swelling + stiffness

344
Q

What are the 5 key subtypes of JIA?

A

systemic
polyarticular
oligoarticular
enthesitis related arthritis
juvenile psoriatic arthritis

345
Q

What are the features of systemic JIA?

A

subtle SALMON-PINK rash
joint inflammation + pain
muscle pain

systemic illness - high swinging fevers, enlarged lymph nodes, weight loss
splenomegaly
pleuritis, pericarditis

346
Q

What serology is systemic JIA a/w?

A

antinuclear abs + RFs neg

raised CRP + ESR, platelets and serum ferritin

347
Q

What is a key complication of systemic JIA? How does it present?

A

macrophage activation syndrome
= severe activation of immune system with huge inflammatory response - life-threatening

acutely unwell child with DIC, anaemia, thrombocytopenia, bleeding, non-blanching rash
LOW ESR

348
Q

What are the differentials for children with a fever for >5 days?

A

Kawasaki’s disease
Still’s disease (systemic JIA)
rheumatic fever
leukaemia

349
Q

What is polyarticular JIA?

A

idopathic inflammatory arthritis in 5+ joints

350
Q

What is the classic presentation of polyarticular JIA?

A

symmetrical
affects small joints of hands and feet + large joints

minimal systemic sx - may be mild fever, anaemia, reduced growth (always mild unlike systemic JIA)

351
Q

What serology is a/w polyarticular JIA?

A

equivalent of RA in adults

most are sero -ve > neg for RF
sero +ve if +ve for RF - tend to be older/have disease more similar to RA in adults

352
Q

What is oligoarticular JIA?

A

involves 4 joints or less

353
Q

How does oligoarticular JIA present?

A

usually affects a single larger joint (usually knee, ankle)
more often in girls <6yrs

no systemic sx

354
Q

What condition is oligoarticular arthritis a/w?

A

anterior uveitis
- referral to opthalmology

355
Q

What serology is a/w oligoarticular JIA?

A

inflammatory markers normal/mildly elevated
antinuclear abs often +ve
RF usually neg

356
Q

What is enthesitis-related arthritis?

A

paeds version of sero neg spondyloarthropathy conditions eg ankylosing spondylitis, psoriatic + reactive arthritis, IBD related arthritis

inflammatory arthritis.in joints + enthesitis

357
Q

What is enthesitis?

A

inflammation at the point which. atendon of. amuscle inserts into a bone

358
Q

What can cause enthesitis? Which children are more at risk of enthesitis-related arthritis?

A

traumatic stress eg repetitive strain in sort
AI inflammatory process

more common in boys >6yrs

359
Q

What is the diagnostic imaging modality for enthesitis-related arthritis?

A

MRI can demonstrate enthesitis
but not if it is AI or not

360
Q

What gene is a/w enthesitis-related arthritis?

A

HLA B27

361
Q

What conditions and their sx are a/w enthesitis-related arthritis?

A

psoriasis - psoriatic plaques, nail pitting
IBD - intermittent diarrhoea, rectal bleeding

anterior uveitis

362
Q

How will enthesitis present? Where are the key areas it occurs?

A

tender to localised palpation

IPJ of hand
wrist
greater trochanter of lateral hip
quadriceps insertion at ASIS
quadriceps/patella tendon insertion on patella
base of achilles
metatarsal heads on base of foot

363
Q

What is juvenile psoriatic arthritis?

A

sero -ve inflammatory arthritis a/w psoriasis

364
Q

What is the pattern of join involvement in juvenile psoriatic arthritis?

A

varies
can be a/symmetrical in several/few small or large joints

365
Q

What signs are likely to be seen in a child with juvenile psoriatic arthritis?

A

plaques of psoriasis
nail pitting
onycholysis - separation of nail from bed
dactylitis - inflammation of the full finger
enthesitis

366
Q

How is JIA managed?

A

paed rheumatology team

depends on severity/response:
- NSAIDS
- steroids - oral, IM, intra-articular in oligoarthritis
- DMARDS eg methotrexate, sulfasalazine
- biologic therapy eg TNFi - etanercept, infliximab, adalimumab

367
Q

How does the foetal heart work in utero?

A

R side has higher pressure
blood shunt from RA > LA through foramen ovale
so most blood bypasses the lungs

1st breath: pulmonary blood flow increases > decrease in RA pressure + increases in LA pressure > foramen ovale closes

368
Q

What medication if taken during pregnancy is a/w congenital heart defects?

A

warfarin

369
Q

What is the GS imaging modality of structural congenital heart abnormalities?

A

echo

370
Q

What are the 5 causes of cyanotic heart defects?

A

tetralogy of fallot
transposition of thegreat vessels
tricuspid atresia
truncus arteriousus

complete AVSD

371
Q

How to distinguish between a cardiac and resp cause fo cyanosis?

A

hypoxic test - give them O2
improvement = resp, no improvement = cardiac

372
Q

What is the pathophysiology of a cyanotic congenital heart disease?

A

R>L shunt - deoxygenated blood returning form body enters L side and goes into the systemic circulation without travelling through the lungs to get oxygenated

373
Q

What are the 4 pathological features of tetralogy of fallot?

A

large VSD
overriding aorta
pulmonary valve stenosis
RV hypertrophy

374
Q

What is an overriding aorta?

A

aortic valve further to the right, above the VSD in ToF

RV contracts + sends blood up > aorta is in the path of that blood (all should go to pulmonary artery) > greater proportion of deoxygenated blood enters the aorta from the RS

375
Q

What features of ToF lead to cyanosis?

A

overriding aorta + pulmonary stenosis (high resistance to flow from RV > encourages blood through VSD into aorta)

encourage shunting of blood from R to L = cyanosis
degreee of cyanosis dependant on severity of pulmonary stenosis

376
Q

What are the RFs for ToF?

A

rubella
increased maternal age
alcohol during pregnancy
diabetic mother

377
Q

How does ToF present on CXR?

A

boot shaped heart due to RV thickening

378
Q

When is ToF likely to be identified?

A

antenatal scans

if not: ejection systolic murmur due to PS during the NIPE
if not: signs of HF as they get older

379
Q

What are the S&S of ToF?

A

CYANOSIS
clubbing
poor feeding and weight gain
ejection systolic murmur over pulmonary valve
tet spells

380
Q

What is a tet spell in ToF? What can cause them? How do they present

A

intermittent symptomatic periods where R > L shunt is temporarily worse

when pulmonary vascular resistance increases/systemic resistance decreases (eg exercise + CO2 increases > vasodilator > systemic resistance decreases > path of least resistance = RV > aorta)

precipitated by waking, exercise, crying

irritable, cyanotic, SoB
can lead to reduced consciousness, seizures, death

381
Q

How are tet spells treated?

A

children may squat/bring knees to chest

supplementary O2
beta blockers (relax RV > improve flow. topulmonary vessels)
IV fluids
morphine
NaHCO3
phenylephrine infusion

382
Q

How are tet spells treated?

A

children may squat/bring knees to chest

supplementary O2
beta blockers (relax RV > improve flow to pulmonary vessels)
IV fluids (increase pre-load > increase vol of blood to pulmonary vessels)
morphine (decrease resp drive > more effective breathing)
NaHCO3 (buffer met acidosis)
phenylephrine infusion (increases systemic vascular resistance)

383
Q

How is ToF managed?

A

prostaglandin infusion in neonates - maintains DA so blood can flow from aorta back to pulmonary arteries

definitive: repair in open heart surgery

384
Q

What is transposition of the great arteries?

A

attachments of the aorta + pulmonary trunk are swapped

RV pumps blood into aorta, LV pumps into the pulmonary vessels
2 separate circulations: systemic and R side, pulmonary and L side

385
Q

TotGA is often a/w with which conditions?

A

VSD
coarctation of the aorta
pulmonary stenosis

386
Q

When does TotGA become life-threatening for a baby?

A

normal development in-utero due to bypassing of lungs

at birth: immediately life-threatening as there is no connection between systemic + pulmonary circulation
= cyanosed

387
Q

How is survival ensured at birth in a baby with TotGA?

A

needs a shunt between systemic and pulmonary circulation so blood can be oxygenated at the lungs

can occur across a patent DA, ASD or VSD

388
Q

How will a baby with TotGA present at birth if not diagnosed antenatally?

A

cyanosed at or within a few days of birth

patent DA or VSD can initially compensate by allowing blood to mix but within a few weeks > resp distress, tachy, poor feeding/weight gain, sweating

389
Q

How is TofGA managed?

A

ASD/VSD delays need for tx

prostaglandin to maintain ductus arteriosus
balloon septostomy - catheter into foramen ovale through umbilicus > inflate balloon to create large ASD

definitive: open heart surgery (cardiopulmonary bypass used to do arterial switch)

390
Q

What is truncus arteriosus?

A

primitive truncus doesn’t divide into pulmonary artery + aorta in development > one blood vessel comes out of. theheart

391
Q

What condition is complete AVSD a/w?

A

Down’s - all need echo at birth

392
Q

What are the non-cyanotic congenital heart defects?

A

VSD
ASD
PDA

L>R shunts

393
Q

How do non-cyanotic heat defects generally present at birth?

A

breathless or asymptomatic
heart murmur
poor feeding, dyspnoea, tachypnoea, failure to thrive

394
Q

Which direction is blood shunted in ASD? What are the consequences of this?

A

L>R as pressure in L is higher

blood flows to pulmonary vessels/lungs to get oxygenated BUT increased flow to R side > R sided overload + strain > risk of R HF + pulmonary HTN

395
Q

What are the possible complications of an ASD?

A

stroke due to VTE
AF/atrial flutter
pulmonary HTN + R HF
eisenmenger

396
Q

What asymptomatic heart condition may be present if a patient has a DVT and then large stroke?

A

ASD

usually DVT > PE
with ASD: clot can go from RA > LA > aorta > brain > large stroke

397
Q

What heart sounds does a child with ASD present with?

A

mid-systolic crescendo-decrescendo murmur loudest at upper L sternal border

fixed split 2nd HS that doesn’t vary with respiration

398
Q

How does ASD present as a child and adult?

A

may be asymptomatic as a child
may have: SoB, difficulty feeding, poor weight gain, LRTIs

or present in adulthood with dyspnoea, HF, stroke

399
Q

How is ASD managed?

A

W&W if small + asymptomatic

surgical correction
anticoag eg aspirin, warfarin to reduce risk of clots/strokes. inadults

400
Q

What is eisenmenger syndrome? What happens. tothe direction of the shunt?

A

pt has a septal defect with L>R shunt (pressure is higher on L = blood still gets oxygenated by the lungs = non-cyanotic)

over time: the extra blood flowing into the R side + lungs increases pressure in pulmonary vessels > pulmonary HTN
when pulmonary pressure > systemic pressure = R>L shunt > deoxygenated blood bypasses lungs to body = CYANOSIS

401
Q

What 3 heart lesions can cause eisenmenger syndrome?

A

ASD
VSD
PDA

402
Q

When in life will eisenmenger develop?

A

1-2yrs with large shunt
in adulthood with small shunts
can develop quickly. inpregnancy

403
Q

How does the bone marrow respond to low ox sats?

A

more rbcs + Hb to increase O2 carrying capacity > polycythaemia (high conc of Hb)

= gives pt a plethoric complexion
+ makes blood more viscous > increases clot risk

404
Q

What examination findings are a/w eisenmenger?

A

pulmonary HTN:
R ventricular heave (RV contracts forcefully against pressure in lungs)
loud P2 (forceful shutting of pulmonary valve)
raised JVP
peripheral oedema

murmur a/w whichever septal defect

R>L shunt/chronic hypoxia:
cyanosis
clubbing
dyspnoea
plethoric complexion (red due to polycythaemia)

405
Q

What are the main causes of death in eisenmenger syndrome?

A

HF
infection
thromboembolism
haemorrhage

406
Q

How is eisenmenger managed?

A

correct underlying defect

once pulmonary pressure high enough to cause syndrome it can’t be medically reversed > definitive = heart-lung transplant

medical mx: O2, sildenafil for pulmonary HTN, tx arrhythmia/polycythaemia = venesection/thrombosis = anticoag/IE = prophylactic abx

407
Q

What genetic conditions is VSD a/w?

A

down’s
turner’s

408
Q

Why are children with VSD acyanotic?

A

increased pressure in LV = L>R shunt > blood still flows through lungs to be oxygenated

leads to R side overloads/HF + increased flow into pulmonary vessels > pulmonary HTN/eisenmenger

409
Q

What murmur is a/w VSD?

A

pan-systolic murmur loudest at L lower sternal border in 3rd/4th ICS
systolic thrill on palpation

410
Q

What are the 3 causes of a pan systolic murmur?

A

VSD (L lower sternal border)
mitral regurg
tricuspid regurg

411
Q

How is VSD managed?

A

W&W if small and asymptomatic
corrected surgically

increased risk of IE > abx prophylaxis

412
Q

When does the ductus arteriosus usually close? Where is it situated?

A

stops functioning within 1-3 days, closes completely within 2-3wks
failure to close = PDA

connection between aorta and pulmonary artery

413
Q

What are the RFs for PDA?

A

PREMATURITY

may be genetic
maternal infections eg rubella

414
Q

Does PDA present in child or adulthood?

A

can be asymptomatic if small - cause no problems and close spontaneously

can present in adulthood with signs of HF

415
Q

Which direction is the shunt in PDA?

A

L>R

pressure higher in aorta than pulmonary artery > L>R shunt > pulmonary HTN > R sided strain > RV hypertrophy > increased flow through pulmonary vessels and returning to L side > LV hypertrophy

416
Q

What murmur is a/w PDA?

A

small one may have none

continuous crescendo-descrendo machinery murmur that continues during the 2nd heart sound making it difficult to hear
loudest beneath L clavicle

417
Q

How is PDA managed?

A

monitored until 1yr
unlikely to close spontaneously after this > surgical/trans-catheter closure
tx earlier if symptomatic

418
Q

What are the features of an innocent murmur?

A

soft
short
systolic
symptomless
situation dependant eg when unwell, quieter standing

419
Q

What are the red flag features of a murmur?

A

loud
diastolic
louder on standing
other sx eg failure. tothrive, feeding difficulty, cyanosis, SoB

420
Q

What are the 3 key ivx to investigate a murmur?

A

ECG
CXR
echo

421
Q

What are the 3 differentials of an ejection systolic murmur?

A

aortic stenosis
pulmonary stenosis
hypertrophic obstructive cardiomyopathy (4th ICS on L sternal border)

422
Q

What is a split 2nd heart sound? When is it pathological?

A

pulmonary valve closes slightly after aortic valve

inspiration: lungs pulled open by chest wall + diaphragm > heart opens = -ve intrathoracic pressure > r side of heart fils faster as it pulls blood from venous system > increased vol in RV = longer for RV to empty in systole > delay in pulmonary valve closing

ie it is normal with inspiration
fixed split sound is the same with insp + exp = occurs in ASD

423
Q

Where are the 4 heart valves located?

A

aortic: 2nd ICS R sternal border
pulmonary: 2nd ICS L sternal border
tricuspid: 5th ICS L sternal border
mitral: 5th ICS mid clavicular line (apex)

424
Q

What is coarctation of the aorta?

A

narrowing of the aortic arch, usually just post-DA
severity varies

425
Q

Which genetic condition is coarctation of the aorta especially a/w?

A

Turner’s

426
Q

Where is pressure increased and decreased in coarctation of the aorta?

A

reduces pressure distal to narrowing
increases pressure proximal to it eg heart, first 3 branches of aorta

427
Q

What are the S&S of coarctation of the aorta?

A

weak femoral pulses = often only neonatal indication
systolic murmur below L clavicle + scapula

tachypnoea, inceased work of breathing
poor feeding
grey floppy baby

over time:
LV heave due to hypertrophy
underdeveloped L arm (reduced flow to L subclavian) + both legs

428
Q

Coarctation of the aorta will impact a 4 limb BP in what way?

A

high BP in limbs supplied by arteries before narrowing
low BP in limbs after narrowing

429
Q

How is coarctation of the aorta managed?

A

mild - may be sx free
severe - may need emergency surgery after birth

emergency:
prostaglandin E keeps DA open while waiting for surgery
corrective surgery + ligation of DA

430
Q

How does aortic valve stenosis present?

A

can be asymptomatic

fatigue
SoB
dizziness, fainting
WORSE ON EXERTION (outflow from LV can’t keep up with demand)
severe: HF

431
Q

What is the murmur a/w aortic stenosis? What other signs are present?

A

ejection systolic murmur at 2nd ICS R sternal border
crescendo-decrescendo
radiates to carotids

ejection click just before murmur
palpable systolic thrill
slow rising pulse + narrow PP

432
Q

How is aortic stenosis managed?

A

regular monitoring (progressive condition) with echos, ECGs, exercise testing
may need to restrict exercise

percutaneous balloon aortic valvoplasty/surgical valvotomy/valve replacement

433
Q

What are the complications of aortic valve stenosis?

A

LV outflow tract obstruction

HF
ventricular arrhythmia
bacterial endocarditis
sudden death, often on exertion

434
Q

Which conditions is pulmonary valve stenosis a/w?

A

ToF
William syndrome
Noonan syndrome
congenital rubella

435
Q

How may pulmonary valve stenosis present?

A

often asymptomatic

severe: fatigue on exertion, SoB, dizziness, fainting

436
Q

What are the heart sounds a/w pulmonary valve stenosis?

A

ejection systolic murmur at the 2nd ICS L sternal border

palpable thrill in same place
RV heave (hypertropy)
raised JVP with giant a waves

437
Q

How is pulmonary valve stenosis managed?

A

W&W often

balloon valvuloplasty
2nd line = open heart surgery

438
Q

What is the most common cause of L sided HF in children?

A

large VSD

439
Q

What are the signs of L versus R HF?

A

L:
SoB, exercise intolerance
palpitations, chest pain
feeding problems, poor/excessive weight gain

R:
peripheral oedema
liver enlargement

440
Q

What is rheumatic fever?

A

AI condition triggered by streptococcus bacteria

Multi-system disorder affecting joints, heart, skin, nervous system

441
Q

What is the pathophysiology of rheumatic fever? What type of hypersensitivity reaction is it?

A

group A beta haemolytic streptococcal (often strep pyogenes that causes tonsilitis)
immune system creates abs against infection > also match antigens on cells of body

= type 2 hypersensitivity reaction around 2-4wks after initial infection

442
Q

How does rheumatic fever present?

A

2-4wks after a streptococcal infection eg tonsilitis

joints: migratory arthritis affecting large joints, red hot swollen painful joints

heart: carditis with pericarditis/myocarditis/endocarditis > tachy or bradycardia, murmurs from valve disease, pericardial rub on ausc, HF

skin: subcut nodules (firm painless nodules on extensor surfaces of joints), erythema marginatum rash (pink rings of varying sizes on torso and proximal limbs

nervous system: chorea (uncontrolled irregular rapid movements of limbs)

443
Q

How is rheumatic fever investigated?

A

throat swab
anti-streptococcal Ab titres
echo, ECG, CXR for heart involvement

444
Q

Which criteria is used to dx rheumatic fever?

A

Jones

445
Q

What are the components of Jones criteria?

A

evidence of recent streptococcal infection +
2 major OR 1 major and 2 minor - JONES-FEAR

major:
Joint arthritis
Organ inflammation eg carditis
Nodules
Erythema marginatum rash
Sydenham chorea

minor:
Fever
ECG changes (prolonged PR) without carditis
Arthralgia without arthritis
Raised inflam markers

446
Q

How is rheumatic fever treated?

A

tx of strep infection helps prevention - phenoxymethylpencillin for tonsilitis

NSAIDS
aspirin + steroids for carditis
prophylactic abx to prevent recurrence/further strep infections

447
Q

What are the complications of rheumatic fever?

A

recurrence
valvular disease, mostly mitral stenosis
chronic HF

448
Q

What is Kawasaki disease?

A

systemic medium-sized vessel vasculitis

449
Q

Which children is Kawasaki disease more likely to occur in?

A

<5yrs
Asian children, particularly Japanese and Korean

450
Q

What are the key features of Kawasaki?

A

persistent high fever (>39) for >5 DAYS

widespread erythematous maculopapular rash + desquamation on palms and soles
STRAWBERRY TONGUE
cracked lips
cervical lympadenopathy
bilateral conjunctivits

any fever >5 days think kawasaki !!

451
Q

What ix are useful to identify Kawasaki?

A

FBC (anaemia, leukocytosis, thrombocytosis)
LFTs (hypoalbuminaemia, elevated liver enzymes)
inflam markers (esp ESR) raised
urinalysis (raised WBCs without infection)
echo (coronary artery pathology)

452
Q

What are the 3 phase of kawasaki?

A

acute: unwell with fever, rash + lymphadenopathy, lasts 1-2wks

subacute: acute sx settle, desquamation + arthralgia occur, risk of coronary artery aneurysm, lasts 2-4wks

convalescent: remaining sx settle, bloods return to normal, cornoary aneursysm may regress, lasts 2-4wks

453
Q

How is Kawasaki treated 1st line?

A

high dose aspirin (tx thrombosis risk)
+ IV IGs (reduce coronary artery aneurysm risk)

454
Q

What is the key concerning complciation of Kawasaki?

A

coronary artery aneurysm

455
Q

What is the condition where aspirin is unusually used in children? Why is aspirin dangerous in children?

A

Kawasaki

risk of Reye’s syndrome > serious liver + brain damage

456
Q

Describe normal development of gross motor skills at 4m-4yrs

A

4m: support head in line with body
6m: started sitting, not unsupported
9m: sit unsupported, crawling, maintain standing position, bounce on legs when supported
12m: stand, cruising
15m: walk unaided
18m: squat and pick things up
2yr: run, kick ball
3yr: climb stairs one foot at a time, stand on one leg for few secs, ride tricycle
4yr: hop, climb and descend stairs normally

457
Q

Describe normal development of fine motor skills at 8wks-5yr

A

8wk: fixes eyes on object 30cm in front and attempts to follow, show preferences for faces over inanimate objects
6m: palmar grasp (wraps thumb and fingers around object)
9m: scissor grasp (squashes between thumb and forefinger)
12m: pincer grasp, scribbles randomly
14m: tower of 2 bricks
18m: clumsily use spoon to eat, tower of 4 bricks
2yr: copies vertical line, tower of 8 bricks, starts to hold pencil with digital pronate grasp rather than first
2.5yr: copies horizontal line, tower of 12 bricks
3yr: copies circle, thread beads onto string
4yr: copies cross an square, can build steps with bricks, cut paper in half with scissors
5yr: copies triangle

458
Q

Describe normal development of language at 3m-4yrs

A

3m: cooing, recognises familiar voices and gets comfort from them
6m: noises with consonants, responds to tone of voice
9m: babbles but no recognisable words
12m: singles words in context, follows simple instructions
18M; 5-10 words, understands nouns
2yrs: combines 2 words, 50+ words total, understands verbs
2.5yrs: combines 3-4 words, understands propositions
3yrs: basic sentences, understands adjectives eg which is bigger
4yrs: tells stories, follows complex instructions

459
Q

Describe normal development of personal/social skills at 6wk-4yrs

A

6wk: smiles
3m: communicates pleasure
6mo: curious, engages with people
9m: cautious with strangers
12m: engage with people by pointing and handling objects, waves bye, claps hands
18m: imitates activities eg using phone
2yr: interest to people other than parents -waves to strangers, plays next to to other children not necessarily with, dry by day
3yr: seek other children to play with, bowel control
4yr: has best friend, dry by night, dresses self, imaginative play

460
Q

When are missed developmental milestones red flags?

A

not able to hold object at 5mo
not sitting unsupported by 12m
not standing independently, no words, no interest in others by 18m
not walking independently at 2yr
not running at 2.5yr

461
Q

Which neonates are likely to develop RDS?

A

born <32wks - before lungs have produced adequate surfactant

462
Q

How may RDS be prevented in babies with suspected/confirmed premature labour?

A

antenatal steroids - increase production of surfactant

463
Q

What are the short term complications of RDS?

A

pneumothorax
infection
apnoea
intraventricular/pulmonary haemorrhage
NEC

464
Q

What are the long term complications of RDS?

A

chronic lung disease of prematurity
retinopathy of prematurity
neurological, hearing and visual impairment

465
Q

Extended hypoxia during birth can lead to what? Why can extended hypoxia happen?

A

contractions > placenta unable to carry out normal gaseous exchange > hypoxia

extended hypoxia > anaerobic resp + bradycardia + reduced consciousness + drop in resp effort

> hypoxic-ischaemic encepthalopathy
can lead to cerebral palsy

466
Q

What are the key principles of neonatal resuscitation?

A
  1. warm/dry baby
  2. calculate APGAR score at 1, 5 and 10 mins
  3. stimulate breathing - dry vigorously, head in neutral position to open airway, check for airway obstruction, consider aspiration under direct visualisation
  4. inflation breaths: neonate gasping/not breathing despite stimulation
    a. 2x 5 inflation breaths of 3 seconds
    b. 30s ventilation breaths
    c. chest compressions with ventilation breaths (if HR <60, 3:1 with ventilation breaths)
  5. consider IV drugs and intubation. near/term babies with possible HIE, consider therapeutic cooling.
467
Q

In inflation breaths during neonatal resuscitation, should air or air + O2 be used?

A

air in term/near term
mix in pre-term

468
Q

What are the 5 components of the APGAR score used in neonatal resuscitation?

A

appearance (skin colour)
pulse
grimmace (response to stimulation)
activity (muscle tone)
respiration

469
Q

Why is delayed umbilical cord clamping used? What are the pros and cons? Which neonates is it used in?

A

large vol of fetal blood in placenta after birth > delayed clamping = time for blood to enter baby’s circulation (placental transfusion)

+ improved Hb, iron stores, BP, reduced intraventricular haemorrhage and NEC
- increases neonatal jaundice

all uncompromised neonates = delay of 1 min
neonates requiring resus = no delay to prevent delay in resus

470
Q

Facial paralysis after birth is a/w with what type of delivery? Is this permanent damage?

A

forceps - damage to facial nerve

function usually returns in a few mo

471
Q

Which nerves are damaged during birth in neonates with Erbs palsy? What births is it a/w?

A

C5/6 nerves in brachial plexus

a/w shoulder dystocia, traumatic/instrumental delivery/large birth weight, fractured clavicle

472
Q

How does Erb’s palsy present at birth?

A

weakness of shoulder abduction + external rotation, arm flexion + finger extension

waiters tip appearance in affected arm: internally rotated shoulder, extended elbow, flexed wrist facing backwards, lack of movement

function usually returns in a few mo

473
Q

Which births are at higher risk of a fractured clavicle?

A

shoulder dystocia
traumatic/instrumental delivery
large birth weight

474
Q

Which causative organism of neonatal sepsis can be transferred from mother to fetus during delivery? How can this be prevented?

A

group B strep

commonly found in vagina
prophylactic abx given during labour to reduce risk of transfer if mother found to have bacteria

475
Q

What are the RFs for neonatal sepsis?

A

vaginal group B strep colonisation
GBS sepsis in previous baby
maternal sepsis, chorioamnionitis, fever >38
prematurity
PROM
prolonged ROM

476
Q

What are the clinical featrues of neonatal sepsis?

A

fever
reduced tone/activity
poor feeding
resp distress/apnoea
vomiting
tachy/bradycardia
hypoxia
jaundice within 24hrs
seizures
hypoglycaemia

have low threshold for tx

477
Q

What investigations should be done when neonatal sepsis is suspected?

A

blood cultures before abx given
baseline FBC + CRP
LP

478
Q

Whata re the 1st line abx for neonatal sepsis?

A

benzylpencillin and gentamycin

lower risk - cefotaxime

479
Q

How is neonatal sepsis managed after abx are given?

A

CRP at 24hrs + 5days
blood cultures at 36hrs
LP if CRP >10

consider stopping abx if clinicall well/cultures neg/CRP <10

480
Q

What are possible causes of HIE in neonates?

A

anything that causes asphyxia

maternal shock
intrapartum haemorrhage
prolapsed/nuchal cord

481
Q

Babies with what clinical features should be suspecetd of having HIE?

A

events that could cause hypoxia
acidosis on umbilical artery blood gas
poor Apgar
features of HIE - poor feeding, irritability, hyper-alert, lethargic, hypotonia, seizures, reduced consciousness, apnoea, flaccid/reduced/absent reflexes
evidence of multi-organ failure

482
Q

How is HIE managed?

A

supportive care - resus + ventilation, circulatory support, nutrition, acid base balance, tx of seizures

therapeutic cooling - can protect brain from hypoxic injury

ongoing assessment of development

483
Q

What is therapeutic cooling?

A

cool to 33-34 degrees with cooling blankets for 72hrs

reduces inflammation and neurone loss after acute hypoxic injury
reduces risk of CP< developmental delay, LD, blindness, death

484
Q

When is a baby considered preterm? When is extreme and very preterm? Which babies are likely to have the worst outcome?

A

<28wks = extreme
28-32 = very
32-37 = mod-late preterm

<500g/24wks = v poor outcome
dramatic improvement in prognosis with each extra week

485
Q

What are RFs for prematurity?

A

social deprivation
smoking/alcohol/drugs
over/underweight mother
maternal co-morbidities
twins
personal/fhx of prematurity

486
Q

What are 2 options used to delay preterm birth? Who are they used in?

A

women with hx of preterm birth/US showing cervical length <25mm before 24wks

  1. vaginal progesterone - suppository in vagina to discourage labour
  2. cervical cerclage - suture in cervix to hold it closed
487
Q

When preterm labour is suspected/confirmed, what interventions can be done to improve outcomes?

A

tocolysis with nifedipine - CCB suppresses labour
maternal corticosteroids (if <35ws to reduce morbidity/mortality)
IV MgSO4 (if <24wks, protect brain)
delayed cord clamping/cord milking

488
Q

What issues can premature noenates have in early life?

A

RDS
hypothermia
hypoglycaemia
poor feeding
apnoea, bradycardia
neonatal jaundice
intraventricular haemorrhage
retinopathy of prematurity
NEC
immature immune system

489
Q

What long-term issues can premature neonates have?

A

chronic lung disease of prematurity
learning/behaviour difficulties
susceptibility to infections
hearing/visual impairment cerebral palsy

490
Q

Why do normal Hb ranges vary significantly in early life?

A

transition from fetal to adult Hb in 1st 6m
child adapts to taking O2 from air rather than placenta

491
Q

What are the causes of anaemia in infants? Which is the most common?

A

physiologic anaemia of infancy = most common

anaemia of prematurity
blood loss
haemolysis = common > haemolytic disease of the newborn (ABO/rhesus incompatibility), hereditary spherocytosis, G6PD deficiency
twin twin transfusion

492
Q

What is physiologic anaemia of infancy? Why does it happen?

A

normal dip in Hb around 6-9wks in ehalthy term babies

high O2 to tissues due to high Hb at birth > -ve feedback > EPO fro kidneys suppressed > reduced Hb production by bone marrow

493
Q

Why are premature babies more prone to anaemia?

A

less time in utero receiving Fe from mother
rbc creation cannot keep up with rapid growth in 1st few weeks
reduced EPO levels
blood tests remove sig portion of circulating volume

494
Q

What are the causes of anaemia in older children? Which is most common?

A

IDA 2ndary to dietary insufficiency = most common
blood loss (most commonly from menstruation, worldwide - helminth infection)

others eg sickle cell, thalassaemia, leukaemia, hereditary spherocytosis

495
Q

What are the causes of microcytic anaemia?

A

TAILS:

Thalassaemia
Anaemia of chronic disease
IDA
Lead poisoning
Sideroblastic anaemia

496
Q

What are the causes of normocytic anaemia?

A

Acute blood loss
Anaemia of chronic disease
Aplastic anaemia

Haemolytic anaemia
Hypothyroidism

497
Q

What are the causes of megaloblastic and normoblastic macrocytic anaemia?

A

megaloblastic (impaired DNA synthesis > does not divide normally > large abnormal cell):
- B12 or folate def

normoblastic:
- alcohol
reticulocytosis (due to haemolytic anaemia/blood loss)
hypothyroidism
liver disease
drugs eg azathioprine

498
Q

What does anaemia + high recticulocytes indicate?

A

active production of rbcs to replace lost cells > anaemia due to haemolysis or blood loss

499
Q

How should anaemia be investigated?

A

FBC - MCV, Hb
blood film
reticulocyte count
ferritin
B12, folate
bilirubin
direct coombs test
Hb electrophoresis

500
Q

How is anaemia managed?

A

tx underlying causes
eg IDA - iron supplements

severe: blood transfusions

501
Q

What is ITP?

A

idiopathic thrombocytopenic purpura

= spontaneous low platelet count causing a non-blanching rash

502
Q

What type of reaction is ITP?

A

type II hypersensitivity

production of abs that target + destroy platelets spontaneously/due to trigger eg virus

503
Q

Which children usually get ITP?

A

<10yrs
hx of recent viral illness

504
Q

How does ITP present?

A

bleeding (gums, epistaxis, menorrhagia)
bruising
petechial or purpuric rash due to bleeding under skin

505
Q

What is a petechial versus purpuric rash?

A

petechiae - pin prick spots (<1mm) of bleeding under skin
purpura - larger spots of bleeding under skin

large area of blood collected (>10mm) = ecchymoses

all non-blanching

506
Q

How is ITP diagnosed?

A

urgent FBC - low platelet count, otherwise normal FBC

507
Q

What are possible causes of low paltelets in children?

A

ITP
heparin induced thrombocytopenia
leukaemia

508
Q

How is ITP tx?

A

depends on how low platelets are - usually no tx required, safety net and explain it is benign and should resolve in 6-8 wks, monitor until normal

active bleeding/platelets <10:
- prednisolone
- IV Ig
- blood transfusion. ifrequired
- platelet transfusions only work temporarily - abs destroy transfused platelets too

advice:
- avoid contact sports, IM injections/LPs, NSAIDs/aspirin/blood thinning meds
- managing nosebleeds
seek help after injury that may cause internal bleeding

509
Q

What. arepossible complications of ITP?

A

chronic ITP
anaemia
intracranial/SA haemorrhage
GI bleeding

510
Q

What is leukaemia?

A

cancer of a particular line of stem cells in the bone marrow > unregulated production of certain types of blood cells

511
Q

What are the 3 types of leukaemia that affect children, from most to least common?

A

acute lymphoblastic (ALL)
acute myeloid (AML)
chronic myeloid (CML) = rare

512
Q

When does the incidence of ALL and AML peak in children?

A

ALL: 2-3yrs
AML: 2yrs

513
Q

What is the pathophysiological process behind leukaemia?

A

genetic mutation in one of the precursor cells in the bone marrow > excessive production of 1 type of abnormal wbc > can lead to suppression of other cell lines which can lead to underproduction of other cell types
= pancytopenia > anaemia + leukopenia + thrombocytopaenia (low rbcs, wbcs, palelets)

514
Q

What is the main environmental RF for leukaemia? What conditions is it a/w?

A

radiation exposure eg abdo XR during pregnancy

Down’s
Kleinfelter
Noonan
Fanconi’s anaemia

515
Q

How does leukaemia present?

A

non specifically

persistent fatigue
unexplained fever
FtT
weight loss, night sweats

pallor (anaemia)
petechiae, abnormal bruising + unexplained bleeding (thrombocytopenia)

abdo pain
lymphadenopathy
bone/joint pain
hepatosplenomegaly

516
Q

How should leukaemia be ix?

A

any child with unexplained petechiae or hepatomegaly

urgent FBC - anaemia, leukopenia, thrombocytopenia, lots of the abnormal WBCs
blood film - blast cells
bone marrow/lymph node biopsy

others for staging eg CXR, CT, LP

517
Q

How is leukaemia primarily tx?

A

chemo

518
Q

What causes thalassaemia? How do alpha and beta thalassaemia differ?

A

autosomal recessive genetic defect in protein chains of Hb

defects in alpha chains > alpha
defects in beta chains > beta

519
Q

What are the clinical features of thalassaemia?

A

microcytic ANAEMIA

fatigue, pallor
jaundice
gallstones
splenomegaly (fragile rbcs > break down more easily > spleen)
poor growth/development
pronounced forehead/malar eminences (bone marrow expands to produce extra rcs to compensate from chronic anaemia) + susceptibility to fractures

520
Q

How is thalassaemia diagnosed?

A

FBC - microcytic anaemia
Hb electrophoresis - globin abnormalities
DNA testing - genetic abnormality

521
Q

What must be monitored in pts with thalassaemia? Why?

A

serum ferritin

due to iron overload as a result of faulty rbcs, recurrent transfusions + increased absorption of Fe in the gut in response to anaemia

tx: limit transfusions, perform iron chelation

522
Q

How is thalassaemia managed?

A

depends on type/severity

monitor + no tx

blood transfusions
more transfusions > iron chelation

splenectomy
bone marrow transplant

523
Q

What are febrile convulsions?

A

seizures in children (6m-5yrs) with a high fever

not caused by epilepsy/neurological pathology

524
Q

What are simple and complex febrile convulsions?

A

simple: generalised tonic clonic, <15 mins, occur once during a single febrile illness

complex: partial/focal seizures, >15 mins, multiple times during same febrile illness

525
Q

How are febrile convulsions dx?

A

neurological pathology must be excluded eg epilepsy, meningitis/encephalitis, SOLs, syncopy, electrolyte abnormalities, trauma

viral/bacterial illness

526
Q

How are febrile convulsions managed?

A

identify + manage underlying infection - analgesia

advice - safe place, recovery position, ambulance if >5mins
1st seizure = always hospital

527
Q

Define allergen

A

antigen (protein) that the immune system recognises as foreign and potentially harmful > immune response

528
Q

Define atopy

A

predisposition to having hypersensitivity reactions to allergens eg eczema, asthma, hayfever

529
Q

What is the skin sensitisation theory of allergy?

A

break in the infant’s skin that allows allergens eg peanut proteins from environment to cross skin+ react with immune system

+ child then does not eat food containing the allergen - GI tract would recognise the allergen as food and inform immune system

530
Q

What is a type 1 hypersensitivity reaction?

A

IgE abs to specific allergen trigger mast cells + basophils to release histamines + cytokines

typical food allergy

531
Q

What is a type 2 hypersensitivity reaction?

A

IgG + IgM abs react to an allergen and activate complement system > direct damage to local cells

eg haemolytic disease of newborn, transfusion reactions

532
Q

What is a type 3 hypersensitivity reaction?

A

immune complexes accumulate + cause damage to local tissues

AI conditions eg SLE, RA, henoch-schonlein purpura

533
Q

What is a type 4 hypersensitivity reaction?

A

cell mediated hypersensitivity reactions caused by T lymphocytes > inappropriately activated causing inflammation + damage to local tissues

eg organ transplant rejection, contact dermatitis

534
Q

What are the 3 main ways of testing for an allergy?

A

skin prick = sensitisation not allergy
RAST (bloods to total + specific IgE) = sensitisation not allergy
food challenge testing = GS

sensitisation testing notoriously unreliable

535
Q

How are allergic reactions treated?

A

antihistamines eg cetirizine
steroids
IM adrenalin in anaphylaxis

536
Q

What type of reaction is allergic rhinitis?

A

IgE-mediated type 1 hypersensitivity

537
Q

What is allergic rhinitis? When does it happen?

A

environmental allergens cause allergic inflammatory response in nasal mucosa

seasonal eg hayfever
perennial eg house dust mite allergy
occupational

538
Q

What are the typical features of allergic rhinitis?

A

runny blocked itchy nose
sneezing
itchy red swollen eyes

pmh/fhx atopy

539
Q

How is allergic rhinitis managed?

A

avoid trigger
oral antihistamines (non-sedating eg cetirizine, sedating eg chlorphenamine - piriton)
nasal corticosteroid sprays eg fluticasone
nasal antihistamines

540
Q

What is CAH caused by a deficiency in? How. isit inherited?

A

21-hydroxylase enzyme
> underproduction of cortisol and aldosterone + overproduction of androgens

AR pattern

541
Q

What is. thepathophysiology of CAH?

A

21-hydroxylase responsible for converting prog into aldosterone + cortisol

defect > extra prog that can’t be converted to aldosterone + cortisol > gets converted to testosterone instead > low aldosterone + cortisol + high testosterone

542
Q

In severe cases of CAH, how many males and females present?

A

females - virilised genitalia + enlarged clitoris

hyponatraemia/glycaemia + hyperkalaemia shortly after birth
> poor feeding, vomiting, dehydration, arrhythmias

543
Q

How do mild cases of CAH present in males and females?

A

sx related to high androgen levels, may present in childhood/after puberty

females: tall, facial hair, amenorrheic, deep voice, early puberty

males: tall, deep, large penis, small testicles, early puberty

skin hyperpigmentation (ant pit > more ACTH due to low cortisol > byproduct = melanocyte stimulating hormone > melanin)

544
Q

How is CAH managed?

A

monitor growth + development
cortisol replacement - usually hydrocortisone
aldosterone replacement - usually fludrocortisone

females with virilised genitals may need corrective surgery

545
Q

Define eczema

A

chronic atopic condition caused by defects in the normal continuity of the skin barrier, leading to inflammation in the skin

546
Q

How is eczema managed?

A

maintenance + mx of flares

maintenance: emollients, soap substitutes, avoid hot water/scratching/scrubbing/soaps/environmental triggers eg changes in temp, diet, washing powder, stress

flares: thicker emollients, topical steroids, wet wraps, treat infections
rarely in severe cases: IV abx, oral steroids

547
Q

What is the role of an emollient in eczema?

A

to maintain skin barrier
some may only need a thin one eg E45, severe cases require a very thick greasy emollient eg 50:50 ointment, hydromol

use as thick as tolerated and required to maintain eczema

548
Q

What is the role of a topical steroid in eczema. Give examples of mild > potent steroids.

A

use weakest steroid for shortest period required to get skin under control
thicker skin = stronger steroid, thin skin eg face, genitals = avoid or use very weak steroid
settle down immune activity in skin + reduce inflammation

mild: hydrocortisone
moderate: eumovate
potent: betnovate
veyr potent: dermovate

549
Q

What are the side effects of topical steroids?

A

thinning of skin > more prone to flares, bruising, tearing, stretch marks, telangiectasia
possible systemic absorption depending on strength/location

balance ricks against risk of poorly controlled eczema

550
Q

What is the most common causative organism of bacterial infections in children with eczema? How is it treated?

A

staph aureus

tx with oral flucloxacillin

551
Q

What is eczema herpeticum?

A

viral skin infection in pts with eczema caused by HSV or VZV
= pts can be very unwell

552
Q

What is Steven-Johnsons syndrome?

A

disproportional immune response causes epidermal necrosis
affecting <10% of the body surface (toxic epidermal necrolysis affects more)

553
Q

What are the causes of SJS?

A

meds: anti-epileptics, abx, allopurinol, NSAIDs
infections: HSV, mycoplasma pneumonia, CMV, HIV
certain HLA genetic types

554
Q

How does SJS present?

A

mild > severe/fatal

initially non-specific > fever, cough, sore throat/mouth/eyes, itchy skin
> purple/red rash that spreads across skin and strats to blister
> few days later: skin breaks away and sheds leaving raw tissue underneath

can affect lips, mucous membranes, eyes, urinary tract, lungs + internal organs too

555
Q

How is SJS managed?

A

medical emergency - admission
nutritional care, antiseptic, analgesia, ophthalmology input

steroids
IGs
immunosuppressants

556
Q

What are the complications of SJS?

A

toxic epidermal necrolysis (>10% of body)
2ndary infection: breaks in skin > 2ndary bacterial infection, cellulitis, sepsis
permanent skin damage: scarring/damage to skin/hair/lungs/genitals
visual complications: sore eyes > severe scarring/blindness

557
Q

What is viral exanthemas?

A

eruptive widespread red rash caused by a viral infection

558
Q

What are the 6 viral causes of viral exanthemas?

A
  1. measles
  2. scarlet fever
  3. rubella
  4. duke’s disease
  5. parvovirus B19
    6: roseola infantum
559
Q

What is roseola infantum? What is it caused by? How does it present?

A

human herpes virus 6

1-2wks after infection, presents with sudden onset high fever lasting 3-5 days then disppears suddenly
coryzal sx, sore throat, lympthadenopathy
rash appears for 1-2 days after fever settles - mild erythematous macular rash across arms, legs, trunk + face, not itchy

complication = febrile convulsions due to high temp

560
Q

What is urticaria?

A

aka hives

small itchy lumps on skin, may be a/w patchy erythematous rash
can be widespread or localised

561
Q

What is the pathophysiolgoy behind urticaria?

A

mast cells in the skin release histamine + pro-inflamm chemicals
acute urticaria eg allergic reaction
chronic idiopathic urticaria eg AI reaction in eg SLE = autoantibodies target mast cells and trigger them to release histamine etc

562
Q

What may trigger acute urticaria?

A

food/meds/animal allergies
contacts with chemicals/latex/stinging nettles
meds
viral infections
insect bites
dermatographism

563
Q

What may trigger chronic inducible urticaria?

A

sunlight
temp change
exercise
strong emotions
hot/cold weather
pressure

564
Q

How is urticaria treated?

A

antihistamines
1st choice for chronic urticaria = fexofenadine

short course for severe flares: consider oral steroids

565
Q

What genetic malformation causes Down’s?

A

trisomy 21 (3 copies of chromosome 21)

566
Q

What are the dysmorphic fx of Down’s?

A

hypotonia
brachycephaly (small head, flat back)
short neck
short stature
flattened face and nose
prominent epicanthic folds (cover medial eye and eyelid)
upward sloping palpebral fissures (gaps between lower and upper eyelid)
single palmar crease

567
Q

What complications is Down’s a/w?

A

LD
recurrent otitis media
deafness
visual: myopia, strabismus, cataracts
hypothyroid
cardiac (1 in 3): ASD, VSD, PDA, ToF
atlantoaxial instability
leukaemia + dementia more common

568
Q

What is the combined test for antenatal Down’s screening?

A

between 11-14wks

nuchal translucency (>6mm)
+ b-HCG (high) + PAPPA (low)

569
Q

What are the triple and quadruple tests for antenatal Down’s screening?

A

triple: 14-20wks
bHCG (high) + AFP (low) + serum oestriol (low)

quadruple: 14-20wks
triple + inhibin-A (high)

570
Q

What are the definitive tests after a high antenatal risk score for Down’s?

A

risk score from combined/triple/quadruple testing

offered amniocentesis or CVS > karyotyping of fetal cells collected

alternative = NIPT (new, test mother’s blood which will contain DNA
fragments of the placental tissue which would represent fetal DNA)

571
Q

What routine follow up ix are needed for children with Down’s?

A

thyroid checks
echo
audiometry
eye checks

572
Q

webbed What genetic malformation causes TUrner’s syndrome?

A

female has a single X chromosome (45XO)

573
Q

What features are a/w Turner’s?

A

*short stature
*webbed neck
high arching palate
downward sloping eye + ptosis
broad chest + *widely spaced nipples
cubitus valgus
underdeveloped ovaries + reduced function
late/incompelte puberty
most women = infertile

*classic 3

574
Q

What conditions are a/w turner’s?

A

recurrent ottitis media + UTIs
coarctation. ofthe aorta
hypothyroid
HTN
obesity + DM
osteoporosis
LDs

575
Q

How is Turner’s managed?

A

GH therapy = prevent short stature
oestrogen + progesterone replacement = can help establish female 2ndary sex characteristics/regulate menstrual cycle/prevent OP
fertility tx

576
Q

What genetic malformation causes Klinefelters?

A

males have additional X chromosome (47XXY)

577
Q

How may males with Klinefelter present?

A

tall
wide hips
gynaecomastia
weak muscles
small testicles
reduced libido
shyness
infertility
subtle LDs - speech & language

578
Q

What injections can improve sx of Klinefleter’s?

A

testosterone

579
Q

What genetic malformation causes Marfan syndrome? How is it inherited?

A

AD condition affecting gene responsible for creating fibrillin (component of connective tissue)

580
Q

How do patient’s with Marfan syndrome present?

A

*tall
long neck, limbs + fingers (arachnodactyly)
high arch palate
*hypermobility
pectus carinatum/excavatum
downward sloping palpable fissures
*murmur (aortic/mitral regurg)

581
Q

What is the key complication to look out for with Marfan’s?

A

valve prolapse
aortic aneurysms

582
Q

What is the genetic malformation causing fragile X syndrome? How is it inherited?

A

mutation in FMR1 gene (codes for gragile x mental retardation proteins ) - role in cognitive development

x-linked - unclear if dom/rec, males always affected, females vary in affect

583
Q

What are the dysmorphic features of fragile x?

A

delay in speech & language
intellectual disability, ADHD, autism
long narrow face, large ears
large testicles after puberty
hypermobile joints
seizures

584
Q

How is Noonan’s inherited?

A

AD
a/w with number of genes

585
Q

What are the features of Noonan’s?

A

short stature
broad forehead
downward sloping eyes + ptosis
HYPERTELORISM (wide space between eyes)
prominent nasolabial folds
low set ears
webbed neck
widely spaced nipples

586
Q

What is the genetic malformation that causes Prader-Willi?

A

loss of functional genes on proximal arm of C15 inherited from the father
due to deletion of that part of the chromosome or when both copes inherited from mother

587
Q

What are the 2 key features of prader-willi?

A

insatiable hunger > obesity
hypotonia > difficulty feeding (treat with GH)

588
Q

What genetic malformation causes angelman syndrome?

A

loss of function of the copy of the UBE3A gene inherited from the mother
due to a deletion on chromosome 15/mutation in this gene/2 copies inherited from father

589
Q

What are the 3 novel features of angelman syndrome?

A

happy demeanour
unusual fascination with water
widely spaced teeth

590
Q

What genetic malformation causes Edward’s syndrome?

A

trisomy 18

very short lifespan

591
Q

What are the causes of global developmental delay?

A

Down’s
fragile X
fetal alcohol
rett
metabolic disorders

592
Q

What are the causes of gross motor delay?

A

cerebral palsy
ataxia
myopathy
spina bifida
visual impairment

593
Q

What are the causes of fine developmental delay?

A

dyspraxia
cerebral palsy
muscular dystrophy
visual impairment

594
Q

What are the causes of language developmental delay?

A

specific social circumstances eg multiple languages, siblings doing all talking
hearing impairment
LD
neglect
autism
cerebral palsy

595
Q

What are the causes of personal + social developmental delay?

A

emotional and social neglect
parenting issues
autism

596
Q

What 3 vaccines are given at 8 weeks old?

A

diphtheria, tetanus, pertussis, polio, Hib, hep B
MenB
rotavirus

597
Q

What 3 vaccines are given at 12 weeks old?

A

diphtheria, tetanus, pertussis, polio, Hib, hep B
pneumococcal (PCV)
rotavirus

598
Q

What 2 vaccines are given. at16 weeks old?

A

diphtheria, tetanus, pertussis, polio, Hib, hep B
MenB

599
Q

What 4 vaccines are given at age 1?

A

Hib + MenC
pneumococcal (PCV)
MMR
MenB

600
Q

What 2 vaccines are given at 3yrs 4 months?

A

diphtheria, tetanus, pertussis, polio
MMR

601
Q

What 3 vaccines are given between 12-14 years?

A

HPV 2 doses
tetanus, diphtheria + polio
MenACWY

602
Q

Is cerebral palsy progressive?

A

no
nature of sx change over time during growth and development

603
Q

What are the ante/peri/postnatal causes of cerebral palsy?

A

antenatal: maternal infections, trauma in pregnancy
perinatal: birth asphyxia, pre-term birth
postnatal: meningitis, severe neonatal jaundice, head injury

604
Q

What are the 4 types of cerebral palsy?

A

spastic: hypertonia, reduced function due to UMN damage

dyskinetic: problems controlling muscles tone and hyper + hypotonia = athetoid movements + oro-motor problems, due to basal ganglia damage

ataxic: problems with coordinated movement due to cerebellum damage

mixed

605
Q

How do children with cerebral palsy present?

A

failure to meet milestones
increased/decreased tone
*hand preference below 18 months
problems with coordination, speech or walking
feeding/swallowing problems
LD

606
Q

Describe 5 gaits and their causes that may present in a child with CP

A

hemi/diplegic: UMN lesion
broad based/ataxic: cerebellar
high stepping: foot drop or LMN lesions

607
Q

When is the heel prick test done? What does it test for?

A

hypothyroidism, PKU, metabolic diseases, cystic fibrosis, medium-chain acyl Co-A dehydrogenase deficiency (MCADD)

608
Q

What is the basic outline of child surveillance from birth?

A

newborn: NIPE, hearing screening

1st mo: heel prick

following mo: health advisor input, GP exam at 6-8wks, immunisations

609
Q

ICS inhalers increase the risk of what? How can the risk be reduced?

A

oral candidiasis

use a spacer
rinse mouth out after use

610
Q

When should a child be admitted with tonsilitis?

A

if unable to swallow food/drink > IV fluids + analgesia

611
Q

What is acute suppurative otitis media?

A

otitis media with purulent fluid in middle ear > see mucopurulent discharge

612
Q

How does acute mastoiditis present?

A

tender boggy swelling behind pinna
loss of post-auricular sulcus
auricular proptosis

613
Q

What is the dermatological name for a port wine stain?

A

naevus flammeus

614
Q

What is a disimpaction regime for faecal impaction?

A
  1. macrogol eg polyethylene glycol (if not tolerated, use osmotic instead) + electrolytes
  2. add stimulant eg senna
615
Q

When are bulk-forming laxatives useful?

A

children with small hard stools if fibre can’t be increased in their diet

616
Q

When does otitis media need to be referred to ENT?

A

> 6 episodes in 12m
persistent OM with effusion for >3m bilaterally/>6m unilaterally

617
Q

How is pyloric stenosis managed?

A

Ramstedt pyloromyotomy - excessive muscle removed

618
Q

How is meckel’s diverticulum mx?

A

wedge excision - only if symptomatic

619
Q

Where is the murmur for coarctation of the aorta heard?

A

on back between scapula

620
Q

Where is the murmur for coarctation of the aorta heard?

A

on back between scapula

621
Q

Which condition is congenital heart block a/w?

A

SLE - linked to the presence of maternal anti-Ro and/or anti-La antibodies

622
Q

How does complete heart block present on an ECG?

A

no coordination between atria + ventricles > complete dissociation between P waves + QRS

623
Q

How may heart block present?

A

may be asymptomatic, may have severe HF

generally fine for first few yrs, then present with syncope

624
Q

What key sign in infants is a/w congestive HF?

A

hepatomegaly

625
Q

What is the first sign of puberty in boys?

A

enlargement of testes >4ml

626
Q

What is the most common cause of arrest in children?

A

resp